Sunteți pe pagina 1din 50

CONSOLIDATED CASE DIGESTS in Political Law Review (1st Batch)

1.) TECSON vs. COMELEC ISSUE: Is Fernando Poe, Jr., the hero of silver screen, and now one
of the main contenders for the presidency, a natural-born Filipino or is
Citizenship is a treasured right conferred on those whom the state he not?
believes are deserving of the privilege.
HELD:
FACTS: Respondent Ronald Allan Kelly Poe, also known as
Fernando Poe, Jr.filed his certificate of candidacy for the position of He is a natural-born Filipino.
President of the Republic of the Philippines under the Koalisyon ng
Nagkakaisang Pilipino (KNP) Party. In his certificate of candidacy, Aristotle, who, sometime in 384 to 322 B.C., described the "citizen" to
FPJ, representing himself to be a natural-born citizen of the refer to a man who shared in the administration of justice and in the
Philippines. holding of an office.

Victorino X. Fornier, petitioner initiated a petition to disqualify FPJ There was no such term as "Philippine citizens" during the Spanish
made a material misrepresentation in his certificate of candidacy by regime but "subjects of Spain" or "Spanish subjects." In church
claiming to be a natural-born Filipino citizen when in truth, according records, the natives were called 'indios', denoting a low regard for the
to Fornier, his parents were foreigners; his mother, Bessie Kelley Poe, inhabitants of the archipelago
was an American, and his father, Allan Poe, was a Spanish national, The Spanish Constitution of 1876 was never extended to the
being the son of Lorenzo Pou, a Spanish subject. Granting, petitioner Philippine Islands
asseverated, that Allan F. Poe was a Filipino citizen, he could not have
transmitted his Filipino citizenship to FPJ, the latter being an It was only the Civil Code of Spain, made effective in this jurisdiction
illegitimate child of an alien mother. Petitioner based the allegation of on 18 December 1889, which came out with the first categorical
the illegitimate birth of respondent on two assertions - first, Allan F. enumeration of who were Spanish citizens. -
Poe contracted a prior marriage to a certain Paulita Gomez before his
(a) Persons born in Spanish territory,
marriage to Bessie Kelley and, second, even if no such prior marriage
had existed, Allan F. Poe, married Bessie Kelly only a year after the (b) Children of a Spanish father or mother, even if they were born
birth of respondent. outside of Spain,
In the hearing before the Third Division of the COMELEC The (c) Foreigners who have obtained naturalization papers,
marriage certificate of Allan F. Poe and Bessie Kelley, the birth
certificate of FPJ, and the death certificate of Lorenzo Pou are (d) Those who, without such papers, may have become domiciled
documents of public record in the custody of a public officer. The inhabitants of any town of the Monarchy.
documents have been submitted in evidence by both contending
The Treaty of Paris was entered into
parties during the proceedings before the COMELEC.COMELEC
dismissed for lack of merit. Under Article IX of the treaty, the civil rights and political status of the
native inhabitants of the territories ceded to the United States would
Petitioner assailed the decision of the COMELEC before this Court
be determined by its Congress

the native inhabitants of the Philippines ceased to be Spanish


subjects. Although they did not become American citizens, they,

1|Page
CONSOLIDATED CASE DIGESTS in Political Law Review (1st Batch)

however, also ceased to be "aliens" under American laws and were (1) Those who are citizens of the Philippine Islands at the time of the
thus issued passports describing them to be citizens of the Philippines adoption of this Constitution
entitled to the protection of the United States.
(2) Those born in the Philippines Islands of foreign parents who,
The term "citizens of the Philippine Islands" appeared for the first time before the adoption of this Constitution, had been elected to public
in the Philippine Bill of 1902, also commonly referred to as the office in the Philippine Islands.
Philippine Organic Act of 1902, the first comprehensive legislation of
the Congress of the United States on the Philippines (3) Those whose fathers are citizens of the Philippines.

Under the organic act, a citizen of the Philippines was one who was (4) Those whose mothers are citizens of the Philippines and upon
an inhabitant of the Philippines, and a Spanish subject on the 11th day reaching the age of majority, elect Philippine citizenship.
of April 1899. The term inhabitant was taken to include 1) a native- (5) Those who are naturalized in accordance with law.
born inhabitant, 2) an inhabitant who was a native of Peninsular Spain,
and 3) an inhabitant who obtained Spanish papers on or before 11 Subsection (4),
April 1899
Wich provided that women would automatically lose their Filipino
Controversy arose on to the status of children born in the Philippines citizenship and acquire that of their foreign husbands, resulted in
from 11 April 1899 to 01 July 1902, during which period no citizenship discriminatory situations that effectively incapacitated the women from
law was extant in the Philippines. That the common law principle of transmitting their Filipino citizenship to their legitimate children and
jus soli, otherwise also known as the principle of territoriality, operative required illegitimate children of Filipino mothers to still elect Filipino
in the United States and England, governed those born in the citizenship upon reaching the age of majority. Seeking to correct this
Philippine Archipelago within that period. anomaly, as well as fully cognizant of the newly found status of Filipino
women as equals to men, the framers of the 1973 Constitution crafted
In 1916, the Philippine Autonomy Act, also known as the Jones Law the provisions of the new Constitution on citizenship to reflect such
restated virtually the provisions of the Philippine Bill of 1902, as so concerns -
amended by the Act of Congress in 1912 -
Section 2 of the same article also further provided that
Under the Jones Law, a native-born inhabitant of the Philippines was
deemed to be a citizen of the Philippines as of 11 April 1899 if he was "A female citizen of the Philippines who marries an alien retains her
1) a subject of Spain on 11 April 1899, 2) residing in the Philippines Philippine citizenship, unless by her act or omission she is deemed,
on said date, and, 3) since that date, not a citizen of some other under the law to have renounced her citizenship."
country.
The 1987 Constitution generally adopted the provisions of the 1973
Whether or not jus soli was a mode of acquiring citizenship, the 1935 Constitution, except for subsection (3) thereof that aimed to correct
Constitution brought to an end to any such link with common law, by the irregular situation generated by the questionable proviso in the
adopting, once and for all, jus sanguinis or blood relationship as being 1935 Constitution.
the basis of Filipino citizenship
Section I, Article IV, 1987 Constitution now provides:
Section 1, Article III, 1935 Constitution. The following are citizens of
The following are citizens of the Philippines:
the Philippines

2|Page
CONSOLIDATED CASE DIGESTS in Political Law Review (1st Batch)

(1) Those who are citizens of the Philippines at the time of the adoption The parents of FPJ were Allen Poe and Bessie Kelley; FPJ was born
of this Constitution. to them on August 20, 1939; Allan F. Poe and Bessie Kelley were
married to each other on September 16, 1940; The father of Allan F.
(2) Those whose fathers or mothers are citizens of the Philippines. Poe was Lorenzo Pou; At the time of his death on September 11,
(3) Those born before January 17, 1973 of Filipino mothers, who elect 1954, Lorenzo Poe was 84 years old; The public documents submitted
Philippine citizenship upon reaching the age of majority; and are deemed trustworthy.

(4) Those who are naturalized in accordance with law. It is safe to assume that Lorenzo Pou’s place of residence at the time
of death was the same as his residence before death in the absence
The term "natural-born citizens," is defined to include "those who are of evidence that would attest otherwise. In that case, Lorenzo Pou
citizens of the Philippines from birth without having to perform any act would have benefited from the “en masse Filipinization” that the
to acquire or perfect their Philippine citizenship Philippine Bill effected in 1902. This citizenship would then extend to
his son Allan F. Poe, FPJ’s father.
The date, month and year of birth of FPJ appeared to be 20 August
1939 during the regime of the 1935 Constitution. Through its history, Lorenzo born sometime in 1870 during the Spanish colonization
four modes of acquiring citizenship - naturalization, jus soli, res period. Fornier argues that Lorenzo was not in the Philippines during
judicata and jus sanguinis the crucial period of 1898 to 1902 but there is no existing record to
attest to that claim.Fornier failed to show that Lorenzo was out of the
With the adoption of the 1935 Constitution jus sanguinis or blood
country during that same time period. Lorenzo’s residence at the time
relationship would now become the primary basis of citizenship by
of death was in San Carlos, Pangasinan.
birth. The earliest established direct ascendant of FPJ was his
paternal grandfather Lorenzo Pou, married to Marta Reyes, the father For proof of filiation or paternity, the mandatory rules of civil law would
of Allan F. Poe. While the record of birth of Lorenzo Pou had not been not apply in this case. The duly notarized declaration by Ruby Kelley
presented in evidence, his death certificate, however, identified him to Mangahas, FPJ’s maternal aunt and sister of his mother Bessie,
be a Filipino, a resident of San Carlos, Pangasinan proving the acts of Allan F. Poe, recognizing his own paternal
relationship with FPJ (living with Bessie and the children in one house
The distinctions between legitimacy and illegitimacy should only
as one family) would be accepted.
remain in the sphere of civil law and should not unduly impinge on the
domain of political law. Fornier argues that the mandatory rules under civil rule should apply
because FPJ was an illegitimate son.In the FPJ case, there was no
Natural born citizen – citizens of the Philippines from birth without
signature of Allan F. Poe in the birth certificate of FPJ. Fornier argues
having to perform any act to acquire or perfect their Philippine
that even if Allan F. Poe were Filipino, Allan’s citizenship would not
citizenship. Citizenship of FPJ in relation to grandfather Lorenzo Pou’s
have been transmitted to FPJ because FPJ was illegitimate.
citizenship and father Allan F. Poe’s citizenship. Allan F. Poe was a
Filipino citizen because his father Lorenzo was also Filipino. FPJ was alleged to be illegitimate because of the bigamous marriage
Conclusions with some degree of certainty to be drawn from the between his parents Allan and Bessie for the reason that Allan
documents presented: allegedly had a prior existing marriage to a certain Paulita Gomez.
The Court held that the veracity of this marriage between Paulita and
Allan is doubtful. Fornier also contended that even if Allan and

3|Page
CONSOLIDATED CASE DIGESTS in Political Law Review (1st Batch)

Bessie’s marriage was not bigamous, FPJ was still illegitimate Filipino of his Philippine citizenship if either of the following
because his parents were married after he was born. circumstances is present:

The pronouncement that an illegitimate child cannot inherit the (a) The Republic of the Philippines has a defensive and/or offensive
father’s citizenship has no textual basis in the Constitution and violates pact of alliance with said foreign country; or
the equal protection clause. For jurisprudence that regarded an
illegitimate child to inherit the mother’s citizenship, it was there to (b) The said foreign country maintains armed forces on Philippine
ensure a Filipino nationality for the child with the assumption that the territory with the consent of the Republic of the
mother would gain custody. Philippines: Provided, That the Filipino citizen concerned, at the time
of rendering said service, or acceptance of said commission, and
The 1935 Constitution applies to FPJ since he was born during that taking the oath of allegiance incident thereto, states that he does so
time period and it states that Filipino citizens include those whose only in connection with his service to said foreign country; And
fathers are citizens of the Philippines. provided, finally, That any Filipino citizen who is rendering service to,
or is commissioned in, the armed forces of a foreign country under any
2.) BENGSON VS HRET of the circumstances mentioned in paragraph (a) or (b), shall not be
FACTS: Respondent Cruz was a natural-born citizen of the Republic of the Philippines during the period of his service to, or
Philippines. He was born in San Clemente, Tarlac, on April 27, 1960, commission in, the armed forces of said country. Upon his discharge
of Filipino parents. The fundamental law then applicable was the 1935 from the service of the said foreign country, he shall be automatically
Constitution. entitled to the full enjoyment of his civil and politically entitled to the
full enjoyment of his civil political rights as a Filipino citizen x x x.
On November 5, 1985, however, respondent Cruz enlisted in the
United States Marine Corps and without the consent of the Republic Whatever doubt that remained regarding his loss of Philippine
of the Philippines, took an oath of allegiance to the United States. As citizenship was erased by his naturalization as a U.S. citizen on June
a Consequence, he lost his Filipino citizenship for under 5, 1990, in connection with his service in the U.S. Marine Corps.
Commonwealth Act No. 63, section 1(4), a Filipino citizen may lose On March 17, 1994, respondent Cruz reacquired his Philippine
his citizenship by, among other, "rendering service to or accepting citizenship through repatriation under Republic Act No. 2630. He ran
commission in the armed forces of a foreign country." Said provision for and was elected as the Representative of the Second District of
of law reads: Pangasinan in the May 11, 1998 elections. He won by a convincing
SECTION 1. How citizenship may be lost. – A Filipino citizen may lose margin of 26,671 votes over petitioner Antonio Bengson III, who was
his citizenship in any of the following ways and/or events: then running for reelection.1âwphi1.nêt

xxx Subsequently, petitioner filed a case for Quo Warranto Ad


Cautelam with respondent House of Representatives Electoral
(4) By rendering services to, or accepting commission in, the armed Tribunal (HRET) claiming that respondent Cruz was not qualified to
of a foreign country: Provided, That the rendering of service to, or the become a member of the House of Representatives since he is not a
acceptance of such commission in, the armed forces of a foreign natural-born citizen as required under Article VI, section 6 of the
country, and the taking of an oath of allegiance incident thereto, with Constitution. Petitioner asserts that respondent Cruz may no longer
the consent of the Republic of the Philippines, shall not divest a be considered a natural-born Filipino since he lost his Philippine

4|Page
CONSOLIDATED CASE DIGESTS in Political Law Review (1st Batch)

citizenship when he swore allegiance to the United States in 1995, executory only after two (2) years from its promulgation when the court
and had to reacquire the same by repatriation. He insists that Article is satisfied that during the intervening period, the applicant has (1) not
citizens are those who are from birth without having to perform any act left the Philippines; (2) has dedicated himself to a lawful calling or
to acquire or perfect such citizenship. profession; (3) has not been convicted of any offense or violation of
Government promulgated rules; or (4) committed any act prejudicial
ISSUE: Whether respondent Cruz, a natural-born Filipino who to the interest of the nation or contrary to any Government announced
became an American citizen, can still be considered a natural-born policies.
Filipino upon his reacquisition of Philippine citizenship.
Filipino citizens who have lost their citizenship may however reacquire
RULING: Yes. the same in the manner provided by law. Commonwealth Act. No.
The 1987 Constitution enumerates who are Filipino citizens as follow: (C.A. No. 63), enumerates the three modes by which Philippine
citizenship may be reacquired by a former citizen: (1) by naturalization,
(1) Those who are citizens of the Philippines at the time of the adoption (2) by repatriation, and (3) by direct act of Congress.
of this Constitution;
Naturalization is mode for both acquisition and reacquisition of
(2) Those whose fathers or mothers are citizens of the Philippines; Philippine citizenship. As a mode of initially acquiring Philippine
citizenship, naturalization is governed by Commonwealth Act No. 473,
(3) Those born before January 17, 1973 of Filipino mother, who elect
as amended. On the other hand, naturalization as a mode for
Philippine citizenship upon reaching the age of majority, and
reacquiring Philippine citizenship is governed by Commonwealth Act
(4) Those who are naturalized in accordance with law. No. 63. Under this law, a former Filipino citizen who wishes to
reacquire Philippine citizenship must possess certain
There are two ways of acquiring citizenship: (1) by birth, and (2) by qualifications and none of the disqualification mentioned in Section 4
naturalization. These ways of acquiring citizenship correspond to the of C.A. 473
two kinds of citizens: the natural-born citizen, and the naturalized
citizen. A person who at the time of his birth is a citizen of a particular Repatriation, on the other hand, may be had under various statutes by
country, is a natural-born citizen thereof. those who lost their citizenship due to: (1) desertion of the armed
forces; services in the armed forces of the allied forces in World War
As defined in the same Constitution, natural-born citizens "are those II; (3) service in the Armed Forces of the United States at any other
citizens of the Philippines from birth without having to perform any act time, (4) marriage of a Filipino woman to an alien; and (5) political
to acquire or perfect his Philippine citizenship." economic necessity.
On the other hand, naturalized citizens are those who have become As distinguished from the lengthy process of naturalization,
Filipino citizens through naturalization, generally under repatriation simply consists of the taking of an oath of allegiance
Commonwealth Act No. 473, otherwise known as the Revised to the Republic of the Philippine and registering said oath in the
Naturalization Law, which repealed the former Naturalization Law (Act Local Civil Registry of the place where the person concerned
No. 2927), and by Republic Act No. 530. To be naturalized, an resides or last resided.
applicant has to prove that he possesses all the qualifications and
none of the disqualification provided by law to become a Filipino Having thus taken the required oath of allegiance to the Republic and
citizen. The decision granting Philippine citizenship becomes having registered the same in the Civil Registry of Magantarem,

5|Page
CONSOLIDATED CASE DIGESTS in Political Law Review (1st Batch)

Pangasinan in accordance with the aforecited provision, respondent 1.) NO. It should be stressed that there is no specific statutory or
Cruz is deemed to have recovered his original status as a natural-born procedural rule which authorizes the direct filing of a petition for
citizen, a status which he acquired at birth as the son of a Filipino declaration of election of Philippine citizenship before the courts.
father. It bears stressing that the act of repatriation allows him Respondent cannot now be allowed to seek the intervention of the
to recover, or return to, his original status before he lost his court to confer upon her Philippine citizenship when clearly she has
Philippine citizenship. failed to validly elect Philippine citizenship.

3.) REPUBLIC vs. NORA FE SAGUN Under our laws, there can be no action or proceeding for the judicial
declaration of the citizenship of an individual. Courts of justice exist for
G.R. No. 187567, February 15, 2012 settlement of justiciable controversies, which imply a given right,
FACTS: Respondent is the legitimate child of Albert S. Chan, a legally demandable and enforceable, an act or omission violative of
Chinese national, and Marta Borromeo, a Filipino citizen. She was said right, and a remedy, granted or sanctioned by law, for said breach
born on August 8, 1959 in Baguio City3 and did not elect Philippine of right. As an incident only of the adjudication of the rights of the
citizenship upon reaching the age of majority. In 1992, at the age of parties to a controversy, the court may pass upon, and make a
33 and after getting married to Alex Sagun, she executed an Oath of pronouncement relative to their status. Otherwise, such a
Allegiance4to the Republic of the Philippines. Said document was pronouncement is beyond judicial power. x x x
notarized by Atty. Cristeta Leung on December 17, 1992, but was not Clearly, it was erroneous for the trial court to make a specific
recorded and registered with the Local Civil Registrar of Baguio City. declaration of respondent’s Filipino citizenship as such
Sometime in September 2005, respondent applied for a Philippine pronouncement was not within the court’s competence.
passport. Her application was denied due to the citizenship of her 2.) NO. Be that as it may, even if we set aside this procedural
father and there being no annotation on her birth certificate that she infirmity, still the trial court’s conclusion that respondent duly elected
has elected Philippine citizenship. Consequently, she sought a judicial Philippine citizenship is erroneous since the records undisputably
declaration of her election of Philippine citizenship and prayed that the show that respondent failed to comply with the legal requirements for
Local Civil Registrar of Baguio City be ordered to annotate the same a valid election. Specifically, respondent had not executed a sworn
on her birth certificate. RTC granted the petition and declared Sagun statement of her election of Philippine citizenship. The only
a Filipino citizen. OSG directly filed the case to SC via petition for documentary evidence submitted by respondent in support of her
review on certiorari. claim of alleged election was her oath of allegiance, executed 12 years
ISSUE/s: after she reached the age of majority, which was unregistered. As
aptly pointed out by the petitioner, even assuming arguendo that
1.) Whether respondent’s petition for declaration of election of respondent’s oath of allegiance suffices, its execution was not within
Philippine citizenship is authorized by the Rules of Court and a reasonable time after respondent attained the age of majority and
jurisprudence; and was not registered with the nearest civil registry as required under
Section 1 of C.A. No. 625. The phrase "reasonable time" has been
2.) Whether the respondent has effectively elected Philippine
interpreted to mean that the election should be made generally within
citizenship in accordance with the procedure prescribed by law.
three (3) years from reaching the age of majority. 27 Moreover, there
RULING:

6|Page
CONSOLIDATED CASE DIGESTS in Political Law Review (1st Batch)

was no satisfactory explanation proffered by respondent for the delay The COMELEC First Division annulled the proclamation of Arnado
and the failure to register with the nearest local civil registry. and consequently directed that the order of succession under Section
44 of the Local Government of 1991 be followed. It ruled that Arnaldo’s
Based on the foregoing circumstances, respondent clearly failed to act of consistently using his US passport after renouncing his US
comply with the procedural requirements for a valid and effective citizenship negated his Affidavit of Renunciation.
election of Philippine citizenship. Respondent cannot assert that the
exercise of suffrage and the participation in election exercises Petitioner Casan Macode Maquiling, another candidate for mayor of
constitutes a positive act of election of Philippine citizenship since the Kauswagan, and who garnered the second highest number of votes
law specifically lays down the requirements for acquisition of in the 2010 elections, intervened in the case. Maquiling argued that
citizenship by election. All that is required of the elector is to execute while the First Division correctly disqualified Arnado, the order of
an affidavit of election of Philippine citizenship and, thereafter, file the succession under Section 44 of the LGC is not applicable in this case.
same with the nearest civil registry. Having failed to comply with the Maquiling claims that due to the cancellation of Arnado’s candidacy
foregoing requirements, respondent’s petition before the trial court and the nullification of the latter’s proclamation, he should be
must be denied. proclaimed as the winner.

WHEREFORE, the petition is GRANTED. The Decision of the ISSUES:


Regional Trial Court is REVERSED and SET ASIDE. The petition for
judicial declaration of election of Philippine citizenship filed by 1. Whether the use of foreign passport after renouncing foreign
respondent Nora Fe Sagun is hereby DISMISSED for lack of merit. citizenship affects one’s qualifications to run for public office?

4.) MAQUILING vs. COMELEC 2. Whether the rule on succession in the LGC will apply.

DOCTRINE: The use of foreign passport after renouncing one’s HELD: 1. YES. The act of using a foreign passport after renouncing
foreign citizenship is a positive and voluntary act of representation as one’s foreign citizenship is fatal to Arnado’s bid for public office.
to one’s nationality and citizenship; it does not divest Filipino By renouncing his foreign citizenship, Arnado was deemed to be
citizenship regained by repatriation but it recants the Oath of solely a Filipino citizen, regardless of the effect of such renunciation
Renunciation required to qualify one to run for an elective position. under the laws of the foreign country. However, this legal presumption
FACTS: Rommel Arnado is a natural born Filipino citizen who lost his does not operate permanently and is open to attack when, after
citizenship upon his naturalization as a citizen of the USA. renouncing the foreign citizenship, the citizen performs positive acts
Subsequently, he availed of the benefits of RA 9225, the Citizenship showing his continue possession of a foreign citizenship.
Retention and Re-acquisition Act of 2003 and ran as Mayor of While the act of using a foreign passport is not one of the acts
Kauswagan, Lanao del Norte in the 2010 local elections. enumerated in Commonwealth Act No. 63 constituting renunciation
Linog C. Balua (Balua), another mayoralty candidate, filed a petition and loss of PH citizenship, it is nevertheless an act which repudiates
to disqualify Arnado, contending that Arnado is a foreigner. It turend the very oath of renunciation required for a former Filipino citizen who
out that Arnado has been using his US Passport in entering and is also a citizen of another country to be qualified to run for a local
departing the Philippines. elective position.

7|Page
CONSOLIDATED CASE DIGESTS in Political Law Review (1st Batch)

The Court agrees with the COMELEC En Banc that such act of using FACTS: In 1974, petitioner became a Canadian citizen by
a forein passport does not divest Arnado of his Filipino citizenship, naturalization. Upon their retirement, petitioner and his wife returned
which he acquired by repatriation. However, by representing himself to the Philippines. Sometime in 2000, they purchased a lot along the
as an American citizen, Arnado voluntarily and effectively reverted to beach in Tambong, Gloria, Oriental Mindor. However, in the year
his earlier status as a dual citizen. Such reversion was not retroactive; 2004, they came to know that the portion where they built their house
it took place the instant Arnado represented himself as an American is public land and part of the salvage zone.
citizen by using his US passport.
On April 12, 2007, petitioner filed a Miscellaneous Lease Application
Thus, by the time he filed his certificate of candidacy, Arnado was a (MLA) over the subject land with the Department of Environment and
dual citizen enjoying the rights and privileges of Filipino and American Natural Resources (DENR) at the Community Environment and
citizenship. He was qualified to vote, but by the express Natural Resources Office (CENRO) in Socorro. In the said application,
disqualification under Sec. 40 (d) of the LGC, he was not qualified to petitioner indicated that he is a Filipino citizen.
run for a local elective position.
Private respondent Editha A. Agbay opposed the application on the
2. NO. The rule on succession under the LGC will not apply. ground that petitioner, a Canadian citizen, is disqualified to own land.
She also filed a criminal complaint for falsification of public documents
The electorate’s awareness of the candidate’s disqualification is not a under Article 172 of the Revised Penal Code against the petitioner.
prerequisite for the disqualification to attach to the candidate. The very
existence of a disqualifying circumstance makes the candidate Meanwhile, on October 11, 2007, while petitioner’s MLA was pending,
ineligible. Knowledge by the electorate of a candidate’s petitioner re-acquired his Filipino citizenship under the provisions of
disqualification is not necessary before a qualified candidate who R.A. 9225 as evidenced by Identification Certificate No. 266-10-07
placed second to a disqualified one can be proclaimed as the winner. issued by the Consulate General of the Philippines (Toronto).
The second-placer in the vote count is actually the first-placer among
the qualified candidates. In his defense, petitioner averred that at the time he filed his
application, he had intended to re-acquire Philippine citizenship and
The disqualifying circumstance surrounding Arnado’s candidacy that he had been assured by a CENRO officer that he could declare
involves his citizenship. It does not involve the commission of election himself as a Filipino. He further alleged that he bought the property
offenses a provided for in the first sentence of Sec. 68 of the Omnibus from the Agbays who misrepresented to him that the subject property
Election Code, the effect of which is to disqualify the individual from was titled land and they have the right and authorityto convey the
continuing as a candidate, or if he has already been elected, from same. The dispute had in fact led to the institution of civil and criminal
holding the office. suits between him and private respondent’s family.

5.) DAVID vs. AGBAY On January 8, 2008, the Office of the Provincial Prosecutor issued its
Resolution finding probable cause to indict petitioner for violation of
G.R. No. 199113, March 18, 2015 Article 172 of the RPC and recommending the filing of the
Justice VIILLARAMA, JR., corresponding information in court. Petitioner challenged the said
resolution in a petition for review he filed before the Department of
Justice (DOJ).

8|Page
CONSOLIDATED CASE DIGESTS in Political Law Review (1st Batch)

On June 3, 2008, the CENRO issued an order rejecting petitioner’s In fine, for those who were naturalized in a foreign country, they
MLA. It ruled that petitioner’s subsequent re-acquisition of Philippine shall be deemed to have re-acquired their Philippine citizenship
citizenship did not cure the defect in his MLA which was void ab initio. which was lost pursuant to CA 63, under which naturalization in a
foreign country is one of the ways by which Philippine citizenship may
Petitioner argued that once a natural-born Filipino citizen who had be lost. As its title declares, R.A. 9225 amends CA 63 by doing away
been naturalized in another country re-acquires his citizenship under with the provision in the old law which takes away Philippine
R.A. 9225, his Filipino citizenship is thus deemed not to have been citizenship from natural-born Filipinos who become naturalized
lost on account of said naturalization citizens of other countries and allowing dual citizenship, and also
ISSUE/S: provides for the procedure for re-acquiring and retaining Philippine
citizenship. In the case of those who became foreign citizens after
Whether the petitioner can be considered a Filipino citizen. R.A. 9225 took effect, they shall retain Philippine citizenship
despite having acquired foreign citizenship provided they took
Whether he can be indicted for falsification for representing himself as
the oath of allegiance under the new law.
a Filipino in his Public Land Application despite his subsequent re-
acquisition of Philippine citizenship under the provisions of R.A. 9225. Considering that petitioner was naturalized as a Canadian citizen prior
to the effectivity of R.A. 9225, he belongs to the first category of
RULING: Yes. He is now a Filipino.
natural-born Filipinos under the first paragraph of Section 3 who lost
R.A. 9225, otherwise known as the “Citizenship Retention and Re- Philippine citizenship by naturalization in a foreign country. As the
acquisition Act of 2003,” was signed into law by President Gloria new law allows dual citizenship, he was able to re-acquire his
Macapagal-Arroyo on August 29, 2003. Sections 2 and 3 of said law Philippine citizenship by taking the required oath of allegiance.
read:
Yes, he can still be indicted for falsification.
SEC. 2. Declaration of Policy.–It is hereby declared the policy of the
For the purpose of determining the citizenship of petitioner at the
State that all Philippine citizens who become citizens of another
time of filing his MLA, it is not necessary to discuss on the
country shall be deemed not to have lost their Philippine citizenship
retroactivity of such reacquisition because R.A. 9225 itself treats
under the conditions of this Act.
those of his category as having already lost Philippine
SEC. 3. Retention of Philippine Citizenship.–Any provision of law to citizenship, in contradistinction to those natural-born Filipinos who
the contrary notwithstanding, natural-born citizens of the Philippines became foreign citizens after R.A. 9225 came into force. In other
who have lost their Philippine citizenship by reason of their words, Section 2 declaring the policy that considers Filipinos who
naturalization as citizens of a foreign country are hereby deemed to became foreign citizens as not to have lost their Philippine citizenship,
have reacquired Philippine citizenship upon taking the following should be read together with Section 3, the second paragraph of which
oath of allegiance to the Republic. clarifies that such policy governs all cases after the new law’s
effectivity.
Natural-born citizens of the Philippines who, after the
effectivity of this Act, become citizens of a foreign country shall Petitioner made the untruthful statement in the MLA, a public
retain their Philippine citizenship upon taking the aforesaid oath. document, that he is a Filipino citizen at the time of the filing of said
application, when in fact he was then still a Canadian citizen. Under
CA 63, the governing law at the time he was naturalized as

9|Page
CONSOLIDATED CASE DIGESTS in Political Law Review (1st Batch)

Canadian citizen, naturalization in a foreign country was among On certiorari, the SC reversed the ruling and held (9-6 votes) that Poe
those ways by which a natural-born citizen loses his Philippine is qualified as a candidate for Presidency. Three justices, however,
citizenship. While he re-acquired Philippine citizenship under R.A. abstained to vote on the natural-born citizenship issue.
9225 six months later, the falsification was already a consummated
act, the said law having no retroactive effect insofar as his dual ISSUE: WON Grace Poe-Llamanzares is a natural-born Filipino
citizenship status is concerned. citizen

6.) POE-LLAMANZARES vs. COMELEC HELD: (Yes! Foundlings are considered by the constitution as
natural born citizens. To think otherwise would result to
(CITIZENSHIP; FOUNDLING) discrimination)

FACTS: In her COC for presidency for the May 2016 elections, Grace To assume otherwise is to accept the absurd, if not the virtually
Poe declared that she is a natural-born citizen and that her impossible, as the norm. In the words of the Solicitor General:
residence in the Philippines up to the day before 9 May 2016
would be 10 years and 11 months counted from 24 May 2005. “To deny full Filipino citizenship to all foundlings and render
them stateless just because there may be a theoretical chance
May 24, 2005 was the day she came to the Philippines after deciding that one among the thousands of these foundlings might be the
to stay in the PH for good. Before that however, and even afterwards, child of not just one, but two, foreigners is downright
she has been going to and fro between US and Philippines. She was discriminatory, irrational, and unjust. It just doesn't make any
born in 1968, found as newborn infant in Iloilo, and was legally sense. Given the statistical certainty - 99.9% - that any child born in
adopted. She immigrated to the US in 1991 and was naturalized as the Philippines would be a natural born citizen, a decision denying
American citizen in 2001. On July 18, 2006, the BI granted her petition foundlings such status is effectively a denial of their birthright. There
declaring that she had reacquired her Filipino citizenship under RA is no reason why this Honorable Court should use an improbable
9225. She registered as a voter and obtained a new Philippine hypothetical to sacrifice the fundamental political rights of an entire
passport. In 2010, before assuming her post as an appointed class of human beings. Your Honor, constitutional interpretation and
chairperson of the MTRCB, she renounced her American the use of common sense are not separate disciplines”.
citizenship to satisfy the RA 9225 requirement. From then on, she
stopped using her American passport. The Solicitor General makes the further point that the framers "worked
to create a just and humane society," that "they were reasonable
Petitions were filed before the COMELEC to deny or cancel her patriots and that it would be unfair to impute upon them a
candidacy on the ground particularly, among others, that she discriminatory intent against foundlings." He exhorts that, given the
cannot be considered a natural-born Filipino citizen since she grave implications of the argument that foundlings are not natural-born
cannot prove that her biological parents or either of them were Filipinos, the Court must search the records of the 1935, 1973 and
Filipinos. The COMELEC en banc cancelled her candidacy on the 1987 Constitutions "for an express intention to deny foundlings the
ground that she is in want of citizenship and residence status of Filipinos. The burden is on those who wish to use the
requirements, and that she committed material constitution to discriminate against foundlings to show that the
misrepresentations in her COC. constitution really intended to take this path to the dark side and
inflict this across the board marginalization."

10 | P a g e
CONSOLIDATED CASE DIGESTS in Political Law Review (1st Batch)

perfect their Philippine citizenship." In the first place, "having to


perform an act" means that the act must be personally done by the
We find no such intent or language permitting discrimination citizen. In this instance, the determination of foundling status is
against foundlings. On the contrary, all three Constitutions done not by the child but by the authorities. Secondly, the object
guarantee the basic right to equal protection of the laws. All of the process is the determination of the whereabouts of the
exhort the State to render social justice. Of special consideration parents, not the citizenship of the child. Lastly, the process is
are several provisions in the present charter: Article II, Section certainly not analogous to naturalization proceedings to acquire
11 which provides that the "State values the dignity of every Philippine citizenship, or the election of such citizenship by one
human person and guarantees full respect for human rights," born of an alien father and a Filipino mother under the 1935
Article XIII, Section 1 which mandates Congress to "give highest Constitution, which is an act to perfect it.
priority to the enactment of measures that protect and enhance
the right of all the people to human dignity, reduce social, In this instance, such issue is moot because there is no dispute
economic, and political inequalities x x x" and Article XV, Section that petitioner is a foundling, as evidenced by a Foundling
3 which requires the State to defend the "right of children to Certificate issued in her favor. The Decree of Adoption issued on
assistance, including proper care and nutrition, and special 13 May 1974, which approved petitioner's adoption by Jesusa Sonora
protection from all forms of neglect, abuse, cruelty, exploitation, Poe and Ronald Allan Kelley Poe, expressly refers to Emiliano and his
and other conditions prejudicial to their development." Certainly, wife, Rosario Militar, as her "foundling parents," hence effectively
these provisions contradict an intent to discriminate against affirming petitioner's status as a foundling.
foundlings on account of their unfortunate status.
That the Philippines is not a party to the 1930 Hague Convention
Domestic laws on adoption also support the principle that foundlings nor to the 1961 Convention on the Reduction of Statelessness,
are Filipinos. These laws do not provide that adoption confers does not mean that their principles are not binding. While the
citizenship upon the adoptee. Rather, the adoptee must be a Filipino Philippines is not a party to the 1930 Hague Convention, it is a
in the first place to be adopted. The most basic of such laws is signatory to the Universal Declaration on Human Rights, Article
Article 15 of the Civil Code which provides that "[l]aws relating 15(1) of which effectively affirms Article 14 of the 1930 Hague
to family rights, duties, status, conditions, legal capacity of Convention. Article 2 of the 1961 "United Nations Convention on
persons are binding on citizens of the Philippines even though the Reduction of Statelessness" merely "gives effect" to Article
living abroad." Adoption deals with status, and a Philippine 15(1) of the UDHR.132 In Razon v. Tagitis, this Court noted that the
adoption court will have jurisdiction only if the adoptee is a Philippines had not signed or ratified the "International Convention for
Filipino. the Protection of All Persons from Enforced Disappearance." Yet, we
ruled that the proscription against enforced disappearances in
Lastly, It has been argued that the process to determine that the child the said convention was nonetheless binding as a "generally
is a foundling leading to the issuance of a foundling certificate under accepted principle of international law."
these laws and the issuance of said certificate are acts to acquire
or perfect Philippine citizenship which make the foundling a Adopting these legal principles from the 1930 Hague Convention
naturalized Filipino at best. This is erroneous. Under Article IV, and the 1961 Convention on Statelessness is rational and
Section 2 "Natural-born citizens are those who are citizens of the reasonable and consistent with the jus sanguinis regime in our
Philippines from birth without having to perform any act to acquire or Constitution. The presumption of natural-born citizenship of

11 | P a g e
CONSOLIDATED CASE DIGESTS in Political Law Review (1st Batch)

foundlings stems from the presumption that their parents are The constitutional proscription on alien ownership of lands of the
nationals of the Philippines. As the empirical data provided by public or private domain was intended to protect lands from falling in
the PSA show, that presumption is at more than 99% and is a the hands of non-Filipinos. In this case, however, there would be no
virtual certainty. more public policy violated since the land is in the hands of Filipinos
qualified to acquire and own such land. "If land is invalidly transferred
In sum, all of the international law conventions and instruments to an alien who subsequently becomes a citizen or transfers it to a
on the matter of nationality of foundlings were designed to citizen, the flaw in the original transaction is considered cured and the
address the plight of a defenseless class which suffers from a title of the transferee is rendered valid.33 Thus, the subsequent
misfortune not of their own making. We cannot be restrictive as transfer of the property to qualified Filipinos may no longer be
to their application if we are a country which calls itself civilized impugned on the basis of the invalidity of the initial transfer. 34 The
and a member of the community of nations. objective of the constitutional provision to keep our lands in Filipino
7.) LEE vs. REPUBLIC hands has been achieved.

FACTS: In 1936, the family of Dinglasan sold a parcel of land to Lee 8.) MULLER vs. MULLER
Liong, a Chinese Citizen, In Roxas City. However, in 1948, the former
owners filed with the Court of First Instance, Capiz an action against DOCTRINE: He who seeks equity must do equity, and he who comes
the heirs of Lee Liong for annulment of sale and recovery of land. The into equity must come with clean hands.
plaintiffs assailed the validity of the sale because of the constitutional
prohibition against aliens acquiring ownership of private agriculture FACTS: Petitioner Elena Buenaventura Muller and respondent
land, including residential, commercial or industrial land. The Supreme Helmut Muller were married in Hamburg, Germany on September 22,
Court ruled in favor of Lee and held that both parties were in pari 1989. The couple resided in Germany at a house owned by
delicto respondent’s parents but decided to move and reside permanently in
the Philippines in 1992. By this time, respondent had inherited the
In 1968, another case was filed by the same former owners against house in Germany from his parents which he sold and used the
Lee and the SC ruled that the case was barred by res judicata. The proceeds for the purchase of a parcel of land in Antipolo, Rizal at the
parcel of land has already passed to the heirs of Lee. A reconstitution cost of P528,000.00 and the construction of a house amounting to
P2,300,000.00. The Antipolo property was registered in the name of
of the title was filed in favor of Lee but the OSG filed for an annulment
petitioner, Elena Buenaventura Muller.
of the judgement. The CA ruled in favor of the OSG.

ISSUE: Whether the parcel of land belongs to the petitioner Due to incompatibilities and respondents alleged womanizing,
drinking, and maltreatment, the spouses eventually separated.
HELD: Yes. Subsequent circumstances militate against escheat
proceedings because the land is now in the hands of Filipinos. The On September 26, 1994, respondent filed a petition for separation of
original vendee, Lee Liong, has since died and the land has been properties before the Regional Trial Court of Quezon City. The court
inherited by his heirs and subsequently their heirs, petitioners herein. granted said petition. It also decreed the separation of properties
Petitioners are Filipino citizens, a fact the Solicitor General does not between them and ordered the equal partition of personal properties
dispute. located within the country, excluding those acquired by gratuitous title
during the marriage. With regard to the Antipolo property, the court
held that it was acquired using paraphernal funds of the respondent.

12 | P a g e
CONSOLIDATED CASE DIGESTS in Political Law Review (1st Batch)

However, it ruled that respondent cannot recover his funds because as a rule will follow the law and will not permit that to be done indirectly
the property was purchased in violation of Section 7, Article XII of the which, because of public policy, cannot be done directly.
Constitution.
9.) MATTHEWS vs. TAYLOR
The respondent elevated the case to the Court of Appeals, which
reversed the decision of the RTC. It held that respondent merely FACTS: On June 30, 1988, respondent Benjamin A. Taylor
prayed for reimbursement for the purchase of the Antipolo property, (Benjamin), a British subject, married Joselyn C. Taylor (Joselyn), a
and not acquisition or transfer of ownership to him. It ordered the 17-year old Filipina. On June 9, 1989, while their marriage was
respondent to REIMBURSE the petitioner the amount of P528,000.00 subsisting, Joselyn bought from Diosa M. Martin a 1,294 square-meter
for the acquisition of the land and the amount of P2,300,000.00 for the lot (Boracay property). The sale was allegedly financed by Benjamin.
construction of the house situated in Antipolo, Rizal. Elena Muller then Joselyn and Benjamin, also using the latter’s funds, constructed
filed a petition for review on certiorari. improvements thereon and eventually converted the property to a
vacation and tourist resort known as the Admiral Ben Bow Inn. All
Issue: Whether or not respondent Helmut Muller is entitled to required permits and licenses for the operation of the resort were
reimbursement. obtained in the name of Ginna Celestino, Joselyn’s sister.

Ruling: No, respondent Helmut Muller is not entitled to However, Benjamin and Joselyn had a falling out, and Joselyn ran
reimbursement. away with Kim Philippsen. On June 8, 1992, Joselyn executed a SPA
in favor of Benjamin, authorizing the latter to maintain, sell, lease, and
Ratio Decidendi: sub-lease and otherwise enter into contract with third parties with
respect to their Boracay property.
There is an express prohibition against foreigners owning land in the On July 20, 1992, Joselyn as lessor and petitioner Philip Matthews as
Philippines. lessee, entered into an Agreement of Lease (Agreement) involving the
Boracay property for a period of 25 years, with an annual rental
Art. XII, Sec. 7 of the 1987 Constitution provides: “Save in cases of of P12,000.00. The agreement was signed by the parties and
hereditary succession, no private lands shall be transferred or executed before a Notary Public. Petitioner thereafter took possession
conveyed except to individuals, corporations, or associations qualified
of the property and renamed the resort as Music Garden
to acquire or hold lands of the public domain.”
Resort.1avvphi1
In the case at bar, the respondent willingly and knowingly bought the Claiming that the Agreement was null and void since it was entered
property despite a constitutional prohibition. And to get away with that into by Joselyn without his (Benjamin’s) consent, Benjamin instituted
constitutional prohibition, he put the property under the name of his an action for Declaration of Nullity of Agreement of Lease with
Filipina wife. He tried to do indirectly what the fundamental law bars Damages against Joselyn and the petitioner. Benjamin claimed that
him to do directly.
his funds were used in the acquisition and improvement of the Boracay
property, and coupled with the fact that he was Joselyn’s husband,
With this, the Supreme Court ruled that respondent cannot seek any transaction involving said property required his consent.
reimbursement on the ground of equity. It has been held that equity

13 | P a g e
CONSOLIDATED CASE DIGESTS in Political Law Review (1st Batch)

RTC DECISION. The RTC rendered a decision declaring the Lease the public domain is reserved only to Filipino citizens or corporations
Agreement null and void. The RTC considered the Boracay property at least sixty percent of the capital of which is owned by Filipinos.
as community property of Benjamin and Joselyn; thus, the consent of
the spouses was necessary to validate any contract involving the The rule is clear and inflexible: aliens are absolutely not allowed to
property. The same was affirmed by the CA. Hence, this petition. acquire public or private lands in the Philippines, save only in
constitutionally recognized exceptions. There is no rule more settled
ISSUE: WON THE LEASE AGREEMENT IS VALID than this constitutional prohibition, as more and more aliens attempt
to circumvent the provision by trying to own lands through another. In
RULING: In fine, we are called upon to determine the validity of an a long line of cases, we have settled issues that directly or indirectly
Agreement of Lease of a parcel of land entered into by a Filipino wife involve the above constitutional provision. We had cases where aliens
without the consent of her British husband. In addressing the matter wanted that a particular property be declared as part of their father’s
before us, we are confronted not only with civil law or conflicts of law estate; that they be reimbursed the funds used in purchasing a
issues, but more importantly, with a constitutional question. property titled in the name of another; that an implied trust be declared
It is undisputed that Joselyn acquired the Boracay property in 1989. in their (aliens’) favor; and that a contract of sale be nullified for their
Said acquisition was evidenced by a Deed of Sale with Joselyn as the lack of consent. In fine, the Agreement of Lease entered into between
vendee. The property was also declared for taxation purposes under Joselyn and petitioner cannot be nullified on the grounds advanced by
her name. When Joselyn leased the property to petitioner, Benjamin Benjamin. Thus, we uphold its validity.
sought the nullification of the contract on two grounds: first, that he 10.) WILLEM BEUMER vs. AVELINA AMORES
was the actual owner of the property since he provided the funds used
in purchasing the same; and second, that Joselyn could not enter into G.R. No. 195670 December 3, 2012
a valid contract involving the subject property without his consent.
FACTS: Petitioner, a Dutch National, and respondent, a Filipina,
The trial and appellate courts both focused on the property relations married in March 29, 1980. After several years, the RTC of Negros
of petitioner and respondent in light of the Civil Code and Family Code Oriental declared the nullity of their marriage on the basis of the
provisions. They, however, failed to observe the applicable former’s psychological incapacity as contemplated in Article 36 of the
constitutional principles, which, in fact, are the more decisive. Family Code.

Section 7, Article XII of the 1987 Constitution states: Save in cases of Consequently, petitioner filed a Petition for Dissolution of Conjugal
hereditary succession, no private lands shall be transferred or Partnership praying for the distribution of their properties claimed to
conveyed except to individuals, corporations, or associations qualified have been acquired during the subsistence of their marriage.
to acquire or hold lands of the public domain.1avvphi1
Respondent then averred that, with the exception of their two (2)
Aliens, whether individuals or corporations, have been disqualified residential houses (Lots 1 and 2142), she and petitioner did not
from acquiring lands of the public domain. Hence, by virtue of the acquire any conjugal properties during their marriage, the truth being
aforecited constitutional provision, they are also disqualified from that she used her own personal money to purchase Lots 1, 2142, 5845
acquiring private lands. The primary purpose of this constitutional and 4 out of her personal funds and Lots 2055-A and 2055-I by way
provision is the conservation of the national patrimony. Our of inheritance. She submitted a joint affidavit executed by her and
fundamental law cannot be any clearer. The right to acquire lands of petitioner attesting to the fact that she purchased Lot 2142 and the

14 | P a g e
CONSOLIDATED CASE DIGESTS in Political Law Review (1st Batch)

improvements thereon using her own money. Accordingly, respondent of such admission, the Court finds no reason why it should not apply
sought the dismissal of the petition for dissolution as well as payment the Muller ruling and accordingly, deny petitioner’s claim for
for attorney’s fees and litigation expenses. reimbursement.

During trial, petitioner testified that while Lots 1, 2142, 5845 and 4 As also explained in Muller, the time-honored principle is that he who
were registered in the name of respondent, these properties were seeks equity must do equity, and he who comes into equity must come
acquired with the money he received from the Dutch government as with clean hands. Conversely stated, he who has done inequity shall
his disability benefit since respondent did not have sufficient income not be accorded equity. Thus, a litigant may be denied relief by a court
to pay for their acquisition. He also claimed that the joint affidavit they of equity on the ground that his conduct has been inequitable, unfair
submitted before the Register of Deeds of Dumaguete City was and dishonest, or fraudulent, or deceitful.
contrary to Article 89 of the Family Code, hence, invalid.
In this case, petitioner’s statements regarding the real source of the
ISSUE/s: Whether or not the petitioner, a Dutch national, may funds used to purchase the subject parcels of land dilute the veracity
claim for reimbursement on the payment made to purchase the of his claims: While admitting to have previously executed a joint
subject properties. affidavit that respondent’s personal funds were used to purchase Lot
1, he likewise claimed that his personal disability funds were used to
RULING: NO. The petition lacks merit. acquire the same. Evidently, these inconsistencies show his
The issue to be resolved is not of first impression. In In Re: Petition untruthfulness. Thus, as petitioner has come before the Court with
For Separation of Property-Elena Buenaventura Muller v. Helmut unclean hands, he is now precluded from seeking any equitable
Muller the Court had already denied a claim for reimbursement of the refuge.
value of purchased parcels of Philippine land instituted by a foreigner In any event, the Court cannot, even on the grounds of equity, grant
Helmut Muller, against his former Filipina spouse, Elena reimbursement to petitioner given that he acquired no right
Buenaventura Muller. It held that Helmut Muller cannot seek whatsoever over the subject properties by virtue of its unconstitutional
reimbursement on the ground of equity where it is clear that he purchase. It is well-established that equity as a rule will follow the law
willingly and knowingly bought the property despite the prohibition and will not permit that to be done indirectly which, because of public
against foreign ownership of Philippine land enshrined under Section policy, cannot be done directly. Surely, a contract that violates the
7, Article XII of the 1987 Philippine Constitution which reads: Constitution and the law is null and void, vests no rights, creates no
Section 7. Save in cases of hereditary succession, no private obligations and produces no legal effect at all. Corollary thereto, under
lands shall be transferred or conveyed except to individuals, Article 1412 of the Civil Code, petitioner cannot have the subject
corporations, or associations qualified to acquire or hold lands properties deeded to him or allow him to recover the money he had
of the public domain. spent for the purchase thereof. The law will not aid either party to an
illegal contract or agreement; it leaves the parties where it finds
Undeniably, petitioner openly admitted that he "is well aware of the them. Indeed, one cannot salvage any rights from an unconstitutional
above-cited constitutional prohibition" and even asseverated that, transaction knowingly entered into.
because of such prohibition, he and respondent registered the subject
properties in the latter’s name. Clearly, petitioner’s actuations showed Neither can the Court grant petitioner’s claim for reimbursement on
his palpable intent to skirt the constitutional prohibition. On the basis the basis of unjust enrichment. Nor would the denial of his claim
amount to an injustice based on his foreign citizenship. Precisely, it is

15 | P a g e
CONSOLIDATED CASE DIGESTS in Political Law Review (1st Batch)

the Constitution itself which demarcates the rights of citizens and non- are the orders and processes enumerated immediately above issued
citizens in owning Philippine land. To be sure, the constitutional ban after the rendition of the default judgment.
against foreigners applies only to ownership of Philippine land
and not to the improvements built thereon, such as the two (2) It is contended that the respondent Judge violated the Constitution
houses standing on Lots 1 and 2142 which were properly and the fundamental rule that government funds are exempt from
declared to be co-owned by the parties subject to partition. execution or garnishment when he caused the issuance of the writ of
Needless to state, the purpose of the prohibition is to conserve execution against the Republic
the national patrimony and it is this policy which the Court is ISSUE: WON the Republic can invoke immunity from suit?
duty-bound to protect.
HELD: It is settled that when the State gives its consent to be sued, it
11.) REPUBLIC vs. HON. VICENTE A. HIDALGO does not there by necessarily consent to an unrestrained execution
FACTS: Mendoza filed a suit with the RTC of Manila for reconveyance against it. Tersely put, when the State waives its immunity, all it does,
and the corresponding declaration of nullity of a deed of sale and title in effect,is to give the other party an opportunity to prove, if it can, that
against the Republic. In her complaint Mendoza essentially alleged the state has a liability.T he functions and public services rendered by
being the owner of the disputed Arlegui property which the Republic the State cannot be allowed to paralyzed or disrupted by the diversion
forcibly dispossessed her of and over which the Register of Deeds of of public funds from their legitimate and specific objects, as
Manila issued TCT No. 118911 in the name of the Republic. appropriated by law

The Republic set up, among other affirmative defenses, the States Accordingly, granting private respondents basic plea for recovery of
immunity from suit. the RTC of Manila dismissed Mendozas complaint. the Arlegui property, which was legally hers all along, and the
In her adverted third amended complaint for recovery and reinstatement of her cancelled certificate of title are legally correct as
reconveyance of the Arlegui property, Mendoza sought the declaration they are morally right. While not exactly convenient because the Office
of nullity of a supposed deed of sale which provided the of the President presently uses it for mix residence and office
instrumentation toward the issuance of TCT No. 118911 in the name purposes, restoring private respondent to her possession of the
of the Republic. And aside from the cancellation of TCT No. 118911, Arlegui property is still legally and physically feasible. For what is
Mendoza also asked for the reinstatement of her TCT No. 118527 before us, after all, is a registered owner of a piece of land who, during
the early days of the martial law regime, lost possession thereof to the
The trial court rendered a judgment by default for Mendoza and Government which appropriated the same for some public use, but
against the Republic. To the trial court, the Republic had veritably without going through the legal process of expropriation, let alone
confiscated Mendozas property, and deprived her not only of the use paying such owner just compensation.
thereof but also denied her of the income she could have had
otherwise realized during all the years she was illegally dispossessed The Court finds the monetary award set forth therein to be erroneous.
of the same. And the error relates to basic fundamentals of law as to constitute
grave abuse of discretion.
Petitioner urges the Court to strike down as a nullity the trial courts
order declaring it in default and the judgment by default that followed. But surely, the Arlegui property cannot possibly be assigned, even
Sought to be nullified, too, also on the ground that they were issued in perhaps at the present real estate business standards, a monthly
grave abuse of discretion amounting to lack or in excess of jurisdiction, rental value of at least P500,000.00 or P6,000,000.00 a year, the

16 | P a g e
CONSOLIDATED CASE DIGESTS in Political Law Review (1st Batch)

amount private respondent particularly sought and attempted to prove. amended by Presidential Decree (P.D.) 984 or the Pollution Control
This asking figure is clearly unconscionable, if not downright Law. Under these statutes, the Pollution Adjudication Board (PAB)
ridiculous, attendant circumstances considered. To the Court, an has primary jurisdiction over pollution cases and actions for related
award of P20,000.00 a month for the use and occupancy of the Arlegui damages.
property, while perhaps a little bit arbitrary, is reasonable and may be
granted pro hac vice Shell also claimed that it could not be sued pursuant to the
doctrine of state immunity without the States consent. Shell said
12.) SHELL PHILIPPINES vs. JALOS that under Service Contract 38, it served merely as an agent of
the Philippine government in the development of the Malampaya
FACTS: Shell Philippines Exploration B.V. (Shell) and the Republic of gas reserves.
the Philippines entered into Service Contract 38 for the exploration
and extraction of petroleum in northwestern Palawan. Two years later, ISSUE: W/N Shell can be sued pursuant to the doctrine of state
Shell discovered natural gas in the Camago-Malampaya area and immunity.
pursued its development of the well under the Malampaya Natural Gas
Project. This entailed the construction and installation of a pipeline RULING: Yes. Shell is not an agent of the Republic of
from Shells production platform to its gas processing plant in the Philippines. It is but a service contractor for the exploration and
Batangas. The pipeline spanned 504 kilometers and crossed development of one of the country’s natural gas reserves. While the
the Oriental Mindoro Sea. Republic appointed Shell as the exclusive party to conduct petroleum
operations in the Camago-Malampayo area under the States full
On May 19, 2003, respondents Efren Jalos, Joven Campang, Arnaldo control and supervision, it does not follow that Shell has become the
Mijares, and 75 other individuals (Jalos, et al) filed a complaint for States agent within the meaning of the law.
damages. against Shell before the Regional Trial Court (RTC), Branch
41, Pinamalayan, Oriental Mindoro. Jalos, et al claimed that they were
all subsistence fishermen from the coastal barangay of Bansud, An agent is a person who binds himself to render some service or to
Oriental Mindoro whose livelihood was adversely affected by the do something in representation or on behalf of another, with the
construction and operation of Shells natural gas pipeline. consent or authority of the latter. The essence of an agency is the
Jalos, et al claimed that their fish catch became few after the agents ability to represent his principal and bring about business
construction of the pipeline. As a result, their average net income per relations between the latter and third persons. An agents ultimate
month fell from a high of P4,848.00 to only P573.00. They said that undertaking is to execute juridical acts that would create, modify or
the pipeline greatly affected biogenically hard-structured communities extinguish relations between his principal and third persons. It is this
such as coral reefs and led [to] stress to the marine life in power to affect the principals contractual relations with third persons
the Mindoro Sea. They now have to stay longer and farther out at sea that differentiates the agent from a service contractor.
to catch fish, as the pipelines operation has driven the fish population Shells main undertaking under Service Contract 38 is to perform all
out of coastal waters. petroleum operations and provide all necessary technology and
Instead of filing an answer, Shell moved for dismissal of the finance as well as other connected services to the Philippine
complaint. It alleged that the trial court had no jurisdiction over the government. As defined under the contract, petroleum operation
action, as it is a pollution case under Republic Act (R.A.) 3931, as means the searching for and obtaining Petroleum within

17 | P a g e
CONSOLIDATED CASE DIGESTS in Political Law Review (1st Batch)

the Philippines, including the transportation, storage, handling and DOCTRINE: When the State does waive its sovereign immunity, it is
sale of petroleum whether for export or domestic consumption. Shells only giving the plaintiff the chance to prove, if it can, that the defendant
primary obligation under the contract is not to represent the Philippine is liable.
government for the purpose of transacting business with third persons.
Rather, its contractual commitment is to develop and manage FACTS: UP entered into an agreement with respondent Stern
petroleum operations on behalf of the State. Builders for the extension and renovation of the CAS Building in UPLB.
Stern submitted three progress billings and UP paid two of them, but
Consequently, Shell is not an agent of the Philippine government, but the third one was not paid. Thus, Stern filed a case against UP in RTC
a provider of services, technology and financing for the Malampaya QC wherein it decided in favor of Stern Builders. UP, however, failed
Natural Gas Project. It is not immune from suit and may be sued for to file a timely notice of appeal. The motion for writ of execution of
claims even without the States consent. Notably, the Philippine Stern Builders was granted. RTC respondent Judge Dizon orders for
government itself recognized that Shell could be sued in relation to the the release of garnished funds of UP. Hence, the sheriff served notices
project. This is evident in the stipulations agreed upon by the parties of garnishment on UP’s depository banks.
under Service Contract 38.
ISSUE: Whether the funds of UP can be subject to garnishment.
Article II, paragraph 8, Annex B of Service Contract 38 states that legal
expenses, including judgments obtained against the Parties or any of HELD: NO. The funds of the UP are government funds that are public
them on account of the Petroleum Operations, can be recovered by in character. They include the income accruing from the use of real
Shell as part of operating expenses to be deducted from gross property ceded to the UP that may be spent only for the attainment of
proceeds. Article II, paragraph 9B of the same document allows a its institutional objectives. Hence, the funds subject of this action could
similar recovery for [a]ll actual expenditures incurred and paid by not be validly made the subject of the RTC’s writ of execution or
CONTRACTOR [Shell] in settlement of any and all losses, claims, garnishment.
damages, judgments, and any other expenses not covered by The adverse judgment rendered against the UP in a suit to which it
insurance, including legal services. This signifies that the State itself had impliedly consented was not immediately enforceable by
acknowledged the suability of Shell. Since payment of claims and execution against the UP, because suability of the State did not
damages pursuant to a judgment against Shell can be deducted from necessarily mean its liability. Suability depends on the consent of the
gross proceeds, the State will not be required to perform any additional State to be sued, liability on the applicable law and the established
affirmative act to satisfy such a judgment. facts. The circumstance that a state is suable does not necessarily
In sum, while the complaint in this case sufficiently alleges a cause of mean that it is liable; on the other hand, it can never be held liable if it
action, the same must be filed with the PAB, which is the government does not first consent to be sued. Liability is not conceded by the mere
agency tasked to adjudicate pollution-related cases. Shell is not an fact that the state has allowed itself to be sued. When the State does
agent of the State and may thus be sued before that body for any waive its sovereign immunity, it is only giving the plaintiff the chance
damages caused by its operations. The parties may appeal the PABs to prove, if it can, that the defendant is liable.
decision to the CA. But pending prior determination by the PAB, courts UP was correct in saying that the garnishment of its funds to satisfy
cannot take cognizance of the complaint. the judgment awards of actual and moral damages (including
13.) UNIVERSITY OF THE PHILIPPINES vs. DIZON attorney’s fees) was not validly made if there was no special
appropriation by Congress to cover the liability.

18 | P a g e
CONSOLIDATED CASE DIGESTS in Political Law Review (1st Batch)

14.) DEPARTMENT OF TRANSPORTATION AND exemplary damages plus attorney's fees and costs of suit, The Court
COMMUNICATIONS (DOTC) vs. SPS. ABECINA of Appeals affirmed the RTC’s decision. Hence this petition.

G.R. No. 206484, June 29, 2016 ISSUE: Whether the DOTC’s contention that instead of allowing
recovery of the property, the case should be remanded to the RTC for
FACTS: Respondent spouses Vicente and Maria Cleofe Abecina determination of just compensation has merit
(respondents/spouses Abecina) are the registered owners of five
parcels of land in Sitio Paltik, Barrio Sta. Rosa, Jose Panganiban, RULING: The Philippines recognizes the vital role of information and
Camarines Norte. The properties are covered by Transfer Certificates communication in nation building. As a consequence, we have
of Title (TCT) Nos. T-25094, T-25095, T-25096, T-25097, and T- adopted a policy environment that aspires for the full development of
25098. communications infrastructure to facilitate the flow of information into,
out of, and across the country. To this end, the DOTC has been
In February 1993, the DOTC awarded Digitel Telecommunications mandated with the promotion, development, and regulation of
Philippines, Inc. (Digitel) a contract for the management, operation, dependable and coordinated networks of communication.
maintenance, and development of a Regional Telecommunications
Development Project (RTDP) under the National Telephone Program, The DOTC encroached on the respondents' properties when it
Phase I, Tranche 1 (NTPI-1) constructed the local telephone exchange in Daet, Camarines Norte.
The exchange was part of the RTDP pursuant to the National
Later on, the municipality of Jose Panganiban, Camarines Norte, Telephone Program. We have no doubt that when the DOTC
donated a one thousand two hundred (1,200) square-meter parcel of constructed the encroaching structures and subsequently entered the
land to the DOTC for the implementation of the RDTP in the FLA with Digitel for their maintenance, it was carrying out a sovereign
municipality. However, the municipality erroneously included portions function. Therefore, we agree with the DOTC's contention that these
of the respondents' property in the donation. Pursuant to the FLAs, are acts jure imperii that fall within the cloak of state immunity.
Digitel constructed a telephone exchange on the property which
encroached on the properties of the respondent spouses The Constitution identifies the limitations to the awesome and near-
limitless powers of the State. Chief among these limitations are the
On April 29, 2003, the respondent spouses sent a final demand letter principles that no person shall be deprived of life, liberty, or property
to both the DOTC and Digitel to vacate the premises and to pay unpaid without due process of law and that private property shall not be taken
rent/damages in the amount of one million two hundred thousand for public use without just compensation. These limitations are
pesos (P1,200,000.00). Neither the DOTC nor Digitel complied with enshrined in no less than the Bill of Rights that guarantees the citizen
the demand. protection from abuse by the State. Consequently, our laws require
The RTC held that as the lawful owners of the properties, the that the State's power of eminent domain shall be exercised through
respondent spouses enjoyed the right to use and to possess them - expropriation proceedings in court. Whenever private property is taken
rights that were violated by the DOTC's unauthorized entry, for public use, it becomes the ministerial duty of the concerned office
construction, and refusal to vacate. The RTC ordered the Department or agency to initiate expropriation proceedings. By necessary
- as a builder in bad faith -to forfeit the improvements and vacate the implication, the filing of a complaint for expropriation is a waiver of
properties; and (2) awarded the spouses with P1,200,000.00 as actual State immunity. If the DOTC had correctly followed the regular
damages, P200,000.00 as moral damages, and P200,000.00 as procedure upon discovering that it had encroached on the
respondents' property, it would have initiated expropriation

19 | P a g e
CONSOLIDATED CASE DIGESTS in Political Law Review (1st Batch)

proceedings instead of insisting on its immunity from suit. The


petitioners would not have had to resort to filing its complaint for  Petitioners claim that sometime prior to the date of expiration
reconveyance. of the said agreement, or before August 1999, they informed
respondent that the renewal of the agreement shall be at the
The exercise of eminent domain requires a genuine necessity to take
the property for public use and the consequent payment of just discretion of the incoming Chief of Administration, Minister
compensation. The property is evidently being used for a public Counsellor Azhari Kasim, who was expected to arrive in
purpose. However, we also note that the respondent spouses willingly February 2000. When Minister Counsellor Kasim assumed
entered into a lease agreement with Digitel for the use of the subject the position of Chief of Administration in March 2000, he
properties. If in the future, the factual circumstances should change allegedly found respondents work and services unsatisfactory
and the respondents refuse to continue the lease, then the DOTC may and not in compliance with the standards set in the
initiate expropriation proceedings. Maintenance Agreement. Hence, the Indonesian Embassy
terminated the agreement in a letter dated August 31, 2000.
15.) REPUBLIC OF INDONESIA vs. VINZON
Petitioners claim, moreover, that they had earlier verbally
G.R. No. 154705. June 26, 2003 informed respondent of their decision to terminate the
agreement.
FACTS: This is a petition for review of the decision made by Court of
 The respondent claims that the aforesaid termination was
Appeals in ruling that the Republic of Indonesia gave its consent to be
arbitrary and unlawful. Hence, he filed a complaint against the
sued and voluntarily submitted itself to the laws and jurisdiction of
petitioners which was opposed by invoking immunity from
Philippine courts and that petitioners Ambassador Soeratmin and
Minister Counsellor Kasim waived their immunity from suit. suit.

 Petitioner, Republic of Indonesia, represented by its ISSUE/S:


Counsellor, Siti Partinah, entered into a Maintenance 1. Whether the Republic of Indonesia can invoke the doctrine of
Agreement in August 1995 with respondent James Vinzon, sovereign immunity from suit.
sole proprietor of Vinzon Trade and Services. The 2. Whether the petitioners Ambassador Soeratmin and Minister
Maintenance Agreement stated that respondent shall, for a Counsellor Kasim may be sued herein in their private
consideration, maintain specified equipment at the Embassy capacities.
Main Building, Embassy Annex Building and the Wisma Duta,
the official residence of petitioner Ambassador Soeratmin. RULING:
 The equipment covered by the Maintenance Agreement are
air conditioning units, generator sets, electrical facilities, water 1. Yes. The Supreme Court ruled that the republic of
heaters, and water motor pumps. It is likewise stated therein Indonesia cannot be deemed to have waived its immunity
that the agreement shall be effective for a period of four years to suit. The mere entering into a contract by a foreign state
and will renew itself automatically unless cancelled by either with a private party cannot be construed as the ultimate test
party by giving thirty days prior written notice from the date of of whether or not it is an act juri imperii or juri gestionis. Such
expiry. act is only the start of the inquiry. There is no dispute that the

20 | P a g e
CONSOLIDATED CASE DIGESTS in Political Law Review (1st Batch)

establishment of a diplomatic mission is an act juri imperii. The FACTS: On 7 September 1971, the governments of the Federal
state may enter into contracts with private entities to maintain Republic of Germany and the Republic of the Philippines ratified an
the premises, furnishings and equipment of the embassy. The Agreement concerning Technical Co-operation (Agreement) in
Republic of Indonesia is acting in pursuit of a sovereign Bonn, capital of what was then West Germany. The Agreement
activity when it entered into a contract with the respondent. affirmed the countries common interest in promoting the technical
The maintenance agreement was entered into by the and economic development of their States, and recogni[zed] the
benefits to be derived by both States from closer technical co-
Republic of Indonesia in the discharge of its governmental
operation, and allowed for the conclusion of arrangements
functions. It cannot be deemed to have waived its immunity
concerning individual projects of technical co-operation.
from suit.
On 10 December 1999, the Philippine government, through then
2. No. Article 31 of the Vienna Convention on Diplomatic Foreign Affairs Secretary Domingo Siazon, and the German
Relations provides that a diplomatic agent shall enjoy government, agreed to an Arrangement in furtherance of the 1971
immunity from the criminal jurisdiction of the receiving Agreement. This Arrangement affirmed the common commitment of
State. He shall also enjoy immunity from its civil and both governments to promote jointly a project called, Social Health
administrative jurisdiction, except in the case of: Insurance Networking and Empowerment (SHINE), which was
 a real action relating to private immovable designed to enable Philippine families especially poor ones to
maintain their health and secure health care of sustainable quality. In
property situated in the territory of the
the arraignment, both governments likewise named their respective
receiving State, unless he holds it on behalf
implementing organizations for SHINE. The Philippines designated
of the sending State for the purposes of the
the Department of Health (DOH) and the Philippine Health Insurance
mission; Corporation (Philhealth) with the implementation of SHINE. For their
 an action relating to succession in which the part, the German government charge[d] the Deustche Gesellschaft
diplomatic agent is involved as executor, fur Technische Zusammenarbeit GmbH, Eschborn, with the
administrator, heir or legatee as a private implementation of its contributions. Private respondents were
person and not on behalf of the sending engaged as contract employees hired by GTZ to work for SHINE
State; on various dates between December of 1998 to September of
 an action relating to any professional or 1999. Due to several disputes with their belgian project manager Anne
commercial activity exercised by the Nicolay, Each of the private respondents received a letter from Nicolay
diplomatic agent in the receiving State dated 11 July 2000, informing them of the pre-termination of their
outside his official functions. contracts of employment on the grounds of serious and gross
insubordination, among others, resulting to loss of confidence and
trust.
16.) DEUTSCHE GESSELSCHAFT FUR TECHNISCHE
ZUSAMMENARBEIT vs. COURT OF APPEALS On 21 August 2000, the private respondents filed a complaint for
illegal dismissal with the NLRC.
(STATE IMMUNITY; AGENT OF THE STATE)

21 | P a g e
CONSOLIDATED CASE DIGESTS in Political Law Review (1st Batch)

On 25 October 2005, GTZ, through counsel, filed a Motion to “An incorporated agency has a charter of its own that invests it
Dismiss, on the ground that the Labor Arbiter had no jurisdiction with a separate juridical personality, like the Social Security
over the case, as its acts were undertaken in the discharge of the System, the University of the Philippines, and the City of Manila. By
governmental functions and sovereign acts of the Government contrast, the unincorporated agency is so called because it has
of the Federal Republic of Germany. no separate juridical personality but is merged in the general
machinery of the government, like the Department of Justice, the
Bureau of Mines and the Government Printing Office.
On 27 November 2000, the Labor Arbiter issued an Order denying the If the agency is incorporated, the test of its suability is found in
Motion to Dismiss. The Order cited, among others, that GTZ was a its charter. The simple rule is that it is suable if its charter says
private corporation which entered into an employment contract; and so, and this is true regardless of the functions it is performing.”
that GTZ had failed to secure from the DFA a certification as to its
diplomatic status. State immunity from suit may be waived by general or special
law. The special law can take the form of the original charter of
ISSUE: WON the complaint for illegal dismissal should have been the incorporated government agency.
dismissed for lack of jurisdiction on account of GTZs insistence that it
enjoys immunity from suit. Now, Is GTZ an incorporated agency of the German government or a
private corporation engaged in the implementation of development
HELD: NO! (GTZ FAILED TO ESTABLISH EVIDENCE THAT THEY projects?
ARE IMMUNE FROM A SUIT FOR FAILURE TO COMPLY WITH
THE RULES IN PIL AND FAILURE TO PRESENT EVIDENCE) GTZs own website elicits that petitioner is federally owned, a federal
enterprise, and founded in 1975 as a company under private law. GTZ
The principle of state immunity from suit, whether a local state or a clearly has a very meaningful relationship with the Federal
foreign state, is reflected in Section 9, Article XVI of the Republic of Germany, which apparently owns it. At the same
Constitution, which states that the State may not be sued without time, it appears that GTZ was actually organized not through a
its consent. legislative public charter, but under private law, in the same way
If the instant suit had been brought directly against the Federal that Philippine corporations can be organized under the
Republic of Germany, there would be no doubt that it is a suit brought Corporation Code even if fully owned by the Philippine
against a State, and the only necessary inquiry is whether said State government.
had consented to be sued. However, the present suit was brought This self-description of GTZ in its own official website gives
against GTZ. It is necessary for us to understand what precisely are further cause for pause in adopting petitioners argument that
the parameters of the legal personality of GTZ. GTZ is entitled to immunity from suit because it is an
Counsel for GTZ characterizes GTZ as the implementing agency implementing agency. The above-quoted statement does not
of the Government of the Federal Republic of Germany. However, dispute the characterization of GTZ as an implementing agency of the
it does not automatically invest GTZ with the ability to invoke Federal Republic of Germany, yet it bolsters the notion that as a
State immunity from suit. The distinction lies in whether the company organized under private law, it has a legal personality
agency is incorporated or unincorporated. The following lucid independent of that of the Federal Republic of Germany.
discussion from Justice Isagani Cruz is pertinent:

22 | P a g e
CONSOLIDATED CASE DIGESTS in Political Law Review (1st Batch)

Taking this description on face value, the apparent equivalent provided factual basis for its claim of immunity that would, at the
under Philippine law is that of a corporation organized under the very least, establish a disputable evidentiary presumption that
Corporation Code but owned by the Philippine government, or a the foreign party is indeed immune which the opposing party will
government-owned or controlled corporation without original have to overcome with its own factual evidence. We do not see
charter. why GTZ could not have secured such certification or endorsement
from the DFA for purposes of this case. Certainly, it would have been
highly prudential for GTZ to obtain the same after the Labor Arbiter
**GTZ has failed to establish that under German law, it has not had denied the motion to dismiss. Still, even at this juncture, we do not
consented to be sued despite it being owned by the Federal see any evidence that the DFA, the office of the executive branch in
Republic of Germany. We adhere to the rule that in the absence charge of our diplomatic relations, has indeed endorsed GTZs claim
of evidence to the contrary, of immunity. It may be possible that GTZ tried, but failed to secure
such certification, due to the same concerns that we have discussed
Ruling in Holy See v. Del Rosario provided a template on how a herein.
foreign entity desiring to invoke State immunity from suit could
duly prove such immunity before our local courts. The principles The Court is thus holds and so rules that GTZ consistently has
enunciated in that case were derived from public international law. been unable to establish with satisfaction that it enjoys the
immunity from suit generally enjoyed by its parent country, the
“In Public International Law, when a state or international agency Federal Republic of Germany.
wishes to plead sovereign or diplomatic immunity in a foreign
court, it requests the Foreign Office of the state where it is sued 17.) ARIGO vs. SWIFT
to convey to the court that said defendant is entitled to immunity. FACTS: In 2013, the USS Guardian of the US Navy ran
In the Philippines, the practice is for the foreign government or the aground on an area near the Tubbataha Reefs, a marine habitat of
international organization to first secure an executive endorsement which entry and certain human activities are prevented and afforded
of its claim of sovereign or diplomatic immunity. But how the protection by a Philippine law. The grounding incident prompted the
Philippine Foreign Office conveys its endorsement to the courts petitioners to seek for issuance of Writ of Kalikasan with TEPO from
varies. In International Catholic Migration Commission v. Calleja, 190 the SC.
SCRA 130 (1990), the Secretary of Foreign Affairs just sent a letter Among those impleaded are US officials in their capacity as
directly to the Secretary of Labor and Employment, informing the latter commanding officers of the US Navy. As petitioners argued, they were
that the respondent-employer could not be sued because it enjoyed impleaded because there was a waiver of immunity from suit between
diplomatic immunity.” US and PH pursuant to the VFA terms.
It is to be recalled that the Labor Arbiter, in both of his rulings, noted Petitioners claimed that the grounding, salvaging and post-salvaging
that it was imperative for petitioners to secure from the Department of operations of the USS Guardian violated their constitutional rights to
Foreign Affairs a certification of respondents diplomatic status and a balanced and healthful ecology since these events caused and
entitlement to diplomatic privileges including immunity from suits.The continue to cause environmental damage of such magnitude as to
requirement might not necessarily be imperative. However, had affect other provinces surrounding the Tubbataha Reefs. Aside from
GTZ obtained such certification from the DFA, it would have damages, they sought a directive from the SC for the institution of civil,

23 | P a g e
CONSOLIDATED CASE DIGESTS in Political Law Review (1st Batch)

administrative and criminal suits for acts committed in violation of international rules on the “traditional uses of the oceans”, which is
environmental laws and regulations in connection with the grounding codified in UNCLOS.
incident. They also prayed for the annulment of some VFA provisions
for being unconstitutional. As to the non-ratification by the US, it must be noted that the US’
refusal to join the UNCLOS was centered on its disagreement with
ISSUE: Whether or not the US Government can be sued in this case UNCLOS’ regime of deep seabed mining (Part XI) which considers the
oceans and deep seabed commonly owned by mankind. Such has
HELD: The general rule on state’s immunity from suit applies in this nothing to do with the acceptance by the US of customary international
case. rules on navigation.
First, any waiver of State immunity under the VFA pertains only Hence, non-membership in the UNCLOS does not mean that the US
to criminal jurisdiction and not to special civil actions such as for the will disregard the rights of the Philippines as a Coastal State over its
issuance of the writ of kalikasan. Hence, contrary to petitioners’ claim, internal waters and territorial sea. It is thus expected of the US to bear
the US government could not be deemed to have waived its immunity “international responsibility” under Art. 31 in connection with the USS
from suit. Guardian grounding which adversely affected the Tubbataha reefs.
Second, the US respondents were sued in their official capacity as 18.) IMBONG vs. OCHOA
commanding officers of the US Navy who have control and
supervision over the USS Guardian and its crew. Since the satisfaction FACTS: Republic Act (R.A.) No. 10354, otherwise known as the
of any judgment against these officials would require remedial actions Responsible Parenthood and Reproductive Health Act of 2012 (RH
and the appropriation of funds by the US government, the suit is Law), was enacted by Congress on December 21, 2012. Challengers
deemed to be one against the US itself. Thus, the principle of State from various sectors of society are questioning the constitutionality of
Immunity – in correlation with the principle of States as sovereign the said Act. The petitioners are assailing the constitutionality of RH
equals “par in parem non habet non imperium” – bars the exercise of Law on the following grounds:
jurisdiction by the court over their persons.
SUBSTANTIAL ISSUES:
However, The US government is liable for damages in relation to the
grounding incident under the customary laws of navigation. 1. The RH Law violates the right to life of the unborn.

The conduct of the US in this case, when its warship entered a 2. The RH Law violates the right to health and the right to protection
restricted area in violation of RA 10067 and caused damage to the against hazardous products.
TRNP reef system, brings the matter within the ambit of Article 31 of 3. The RH Law violates the right to religious freedom.
the UNCLOS. While historically, warships enjoy sovereign immunity
from suit as extensions of their flag State, Art. 31 of the UNCLOS 4. The RH Law violates the constitutional provision on involuntary
creates an exception to this rule in cases where they fail to comply servitude.
with the rules and regulations of the coastal State regarding passage
5. The RH Law violates the right to equal protection of the law.
through the latter’s internal waters and the territorial sea.
6. The RH Law violates the right to free speech.
Although the US to date has not ratified the UNCLOS, as a matter of
long-standing policy, the US considers itself bound by customary

24 | P a g e
CONSOLIDATED CASE DIGESTS in Political Law Review (1st Batch)

7. The RH Law is “void-for-vagueness” in violation of the due process 8. Prohibition against involuntary servitude
clause of the Constitution.
PROCEDURAL:
8. The RH Law intrudes into the zone of privacy of one’s family
protected by the Constitution Whether the Court can exercise its power of judicial review over the
controversy.
PROCEDURAL:
1. Actual Case or Controversy
Whether the Court may exercise its power of judicial review over the
controversy. 2. Facial Challenge

1. Power of Judicial Review 3. Locus Standi

2. Actual Case or Controversy 4. Declaratory Relief

3. Facial Challenge 5. One Subject/One Title Rule

4. Locus Standi Discussions:

5. Declaratory Relief PROCEDURAL

6. One Subject/One Title Rule Judicial Review Jurisprudence is replete with the rule that the power
of judicial review is limited by four exacting requisites: (a) there must
Issue/s: be an actual case or controversy; (b) the petitioners must possess
locus standi; (c) the question of constitutionality must be raised at the
SUBSTANTIAL ISSUES: earliest opportunity; and (d) the issue of constitutionality must be the
Whether or not (WON) RA 10354/Reproductive Health (RH) Law is lis mota of the case.
unconstitutional for violating the: Actual Controversy: An actual case or controversy means an
1. Right to life existing case or controversy that is appropriate or ripe for
determination, not conjectural or anticipatory, lest the decision of the
2. Right to health court would amount to an advisory opinion. It must concern a real,
tangible and not merely a theoretical question or issue. There ought
3. Freedom of religion and right to free speech
to be an actual and substantial controversy admitting of specific relief
4. Right to privacy (marital privacy and autonomy) through a decree conclusive in nature, as distinguished from an
opinion advising what the law would be upon a hypothetical state of
5. Freedom of expression and academic freedom facts. Corollary to the requirement of an actual case or controversy is
the requirement of ripeness. A question is ripe for adjudication when
6. Due process clause
the act being challenged has had a direct adverse effect on the
7. Equal protection clause individual challenging it. For a case to be considered ripe for
adjudication, it is a prerequisite that something has then been

25 | P a g e
CONSOLIDATED CASE DIGESTS in Political Law Review (1st Batch)

accomplished or performed by either branch before a court may come interested are informed of the nature, scope and consequences of the
into the picture, and the petitioner must allege the existence of an proposed law and its operation. Moreover, this Court has invariably
immediate or threatened injury to himself as a result of the challenged adopted a liberal rather than technical construction of the rule “so as
action. He must show that he has sustained or is immediately in not to cripple or impede legislation.” The one subject/one title rule
danger of sustaining some direct injury as a result of the act expresses the principle that the title of a law must not be “so uncertain
complained of that the average person reading it would not be informed of the
purpose of the enactment or put on inquiry as to its contents, or which
Facial Challenge: A facial challenge, also known as a First is misleading, either in referring to or indicating one subject where
Amendment Challenge, is one that is launched to assail the validity of another or different one is really embraced in the act, or in omitting
statutes concerning not only protected speech, but also all other rights any expression or indication of the real subject or scope of the act.”
in the First Amendment. These include religious freedom, freedom of
the press, and the right of the people to peaceably assemble, and to Declaration of Unconstitutionality: Orthodox view: An
petition the Government for a redress of grievances. After all, the unconstitutional act is not a law; it confers no rights; it imposes no
fundamental right to religious freedom, freedom of the press and duties; it affords no protection; it creates no office; it is, in legal
peaceful assembly are but component rights of the right to one’s contemplation, as inoperative as though it had never been passed.
freedom of expression, as they are modes which one’s thoughts are Modern view: Under this view, the court in passing upon the question
externalized. of constitutionality does not annul or repeal the statute if it finds it in
conflict with the Constitution. It simply refuses to recognize it and
Locus Standi: Locus standi or legal standing is defined as a personal determines the rights of the parties just as if such statute had no
and substantial interest in a case such that the party has sustained or existence. But certain legal effects of the statute prior to its declaration
will sustain direct injury as a result of the challenged governmental act. of unconstitutionality may be recognized. Requisites for partial
It requires a personal stake in the outcome of the controversy as to unconstitutionality: (1) The Legislature must be willing to retain the
assure the concrete adverseness which sharpens the presentation of valid portion(s), usually shown by the presence of a separability clause
issues upon which the court so largely depends for illumination of in the law; and (2) The valid portion can stand independently as law.
difficult constitutional questions.
Ruling/s:
Transcendental Importance: the Court leans on the doctrine that
“the rule on standing is a matter of procedure, hence, can be relaxed SUBSTANTIAL
for non-traditional plaintiffs like ordinary citizens, taxpayers, and
legislators when the public interest so requires, such as when the 1. Majority of the Members of the Court believe that the question
matter is of transcendental importance, of overreaching significance of when life begins is a scientific and medical issue that should
to society, or of paramount public interest.” not be decided, at this stage, without proper hearing and
evidence. However, they agreed that individual Members
One Subject-One Title: The “one title-one subject” rule does not could express their own views on this matter.
require the Congress to employ in the title of the enactment language
of such precision as to mirror, fully index or catalogue all the contents Article II, Section 12 of the Constitution states: “The State recognizes
and the minute details therein. The rule is sufficiently complied with if the sanctity of family life and shall protect and strengthen the family
the title is comprehensive enough as to include the general object as a basic autonomous social institution. It shall equally protect the life
which the statute seeks to effect, and where, as here, the persons of the mother and the life of the unborn from conception.”

26 | P a g e
CONSOLIDATED CASE DIGESTS in Political Law Review (1st Batch)

In its plain and ordinary meaning (a canon in statutory construction), 2. The RH Law does not intend to do away with RA 4729 (1966).
the traditional meaning of “conception” according to reputable With RA 4729 in place, the Court believes adequate
dictionaries cited by the ponente is that life begins at fertilization. safeguards exist to ensure that only safe contraceptives are
Medical sources also support the view that conception begins at made available to the public. In fulfilling its mandate under
fertilization. Sec. 10 of the RH Law, the DOH must keep in mind the
The framers of the Constitution also intended for (a) “conception” to provisions of RA 4729: the contraceptives it will procure shall
refer to the moment of “fertilization” and (b) the protection of the be from a duly licensed drug store or pharmaceutical company
unborn child upon fertilization. In addition, they did not intend to ban and that the actual distribution of these contraceptive drugs
all contraceptives for being unconstitutional; only those that kill or and devices will be done following a prescription of a qualified
destroy the fertilized ovum would be prohibited. Contraceptives that medical practitioner.
actually prevent the union of the male sperm and female ovum, and
those that similarly take action before fertilization should be deemed Meanwhile, the requirement of Section 9 of the RH Law is to be
non-abortive, and thus constitutionally permissible. considered “mandatory” only after these devices and materials have
been tested, evaluated and approved by the FDA. Congress cannot
The intent of the framers of the Constitution for protecting the life of determine that contraceptives are “safe, legal, non-abortificient and
the unborn child was to prevent the Legislature from passing a effective”.
measure prevent abortion. The Court cannot interpret this otherwise.
The RH Law is in line with this intent and actually prohibits abortion. 3. The Court cannot determine whether or not the use of
By using the word “or” in defining abortifacient (Section 4(a)), the RH contraceptives or participation in support of modern RH
Law prohibits not only drugs or devices that prevent implantation but measures (a) is moral from a religious standpoint; or, (b) right
also those that induce abortion and induce the destruction of a fetus or wrong according to one’s dogma or belief. However, the
inside the mother’s womb. The RH Law recognizes that the fertilized Court has the authority to determine whether or not the RH
ovum already has life and that the State has a bounded duty to protect Law contravenes the Constitutional guarantee of religious
it. freedom.
However, the authors of the IRR gravely abused their office when they
The State may pursue its legitimate secular objectives without being
redefined the meaning of abortifacient by using the term “primarily”.
dictated upon the policies of any one religion. To allow religious sects
Recognizing as abortifacients only those that “primarily induce
to dictate policy or restrict other groups would violate Article III, Section
abortion or the destruction of a fetus inside the mother’s womb or the
5 of the Constitution or the Establishment Clause. This would cause
prevention of the fertilized ovum to reach and be implanted in the
the State to adhere to a particular religion, and thus, establishes a
mother’s womb” (Sec. 3.01(a) of the IRR) would pave the way for the
state religion. Thus, the State can enhance its population control
approval of contraceptives that may harm or destroy the life of the
program through the RH Law even if the promotion of contraceptive
unborn from conception/fertilization. This violates Section 12, Article II
use is contrary to the religious beliefs of e.g. the petitioners.
of the Constitution. For the same reason, the definition of
contraceptives under the IRR (Sec 3.01(j)), which also uses the term 4. Section 23A (2)(i) of the RH Law, which permits RH
“primarily”, must be struck down. procedures even with only the consent of the spouse
undergoing the provision (disregarding spousal content),

27 | P a g e
CONSOLIDATED CASE DIGESTS in Political Law Review (1st Batch)

intrudes into martial privacy and autonomy and goes against their participation in the RH education program, the Court
the constitutional safeguards for the family as the basic social reserves its judgment should an actual case be filed before it.
institution. Particularly, Section 3, Article XV of the
Constitution mandates the State to defend: (a) the right of Any attack on its constitutionality is premature because the
spouses to found a family in accordance with their religious Department of Education has not yet formulated a curriculum on age-
convictions and the demands of responsible parenthood and appropriate reproductive health education.
(b) the right of families or family associations to participate in Section 12, Article II of the Constitution places more importance on
the planning and implementation of policies and programs the role of parents in the development of their children with the use of
that affect them. The RH Law cannot infringe upon this mutual the term “primary”. The right of parents in upbringing their youth is
decision-making, and endanger the institutions of marriage superior to that of the State.
and the family.
The provisions of Section 14 of the RH Law and corresponding
The exclusion of parental consent in cases where a minor undergoing provisions of the IRR supplement (rather than supplant) the right and
a procedure is already a parent or has had a miscarriage (Section 7 duties of the parents in the moral development of their children.
of the RH Law) is also anti-family and violates Article II, Section 12 of By incorporating parent-teacher-community associations, school
the Constitution, which states: “The natural and primary right and duty officials, and other interest groups in developing the mandatory RH
of parents in the rearing of the youth for civic efficiency and the program, it could very well be said that the program will be in line with
development of moral character shall receive the support of the the religious beliefs of the petitioners.
Government.” In addition, the portion of Section 23(a)(ii) which reads
“in the case of minors, the written consent of parents or legal guardian 6. The RH Law does not violate the due process clause of the
or, in their absence, persons exercising parental authority or next-of- Constitution as the definitions of several terms as observed
kin shall be required only in elective surgical procedures” is invalid as by the petitioners are not vague.
it denies the right of parental authority in cases where what is involved
is “non-surgical procedures.” The definition of “private health care service provider” must be seen in
relation to Section 4(n) of the RH Law which defines a “public health
However, a minor may receive information (as opposed to service provider”. The “private health care institution” cited under
procedures) about family planning services. Parents are not deprived Section 7 should be seen as synonymous to “private health care
of parental guidance and control over their minor child in this situation service provider.
and may assist her in deciding whether to accept or reject the
information received. In addition, an exception may be made in life- The terms “service” and “methods” are also broad enough to include
threatening procedures. providing of information and rendering of medical procedures. Thus,
hospitals operated by religious groups are exempted from rendering
5. The Court declined to rule on the constitutionality of Section RH service and modern family planning methods (as provided for by
14 of the RH Law, which mandates the State to provide Age- Section 7 of the RH Law) as well as from giving RH information and
and Development-Appropriate Reproductive Health procedures.
Education. Although educators might raise their objection to

28 | P a g e
CONSOLIDATED CASE DIGESTS in Political Law Review (1st Batch)

The RH Law also defines “incorrect information”. Used together in providers to render pro bono Besides the PhilHealth
relation to Section 23 (a)(1), the terms “incorrect” and “knowingly” accreditation, no penalty is imposed should they do otherwise.
connote a sense of malice and ill motive to mislead or misrepresent
the public as to the nature and effect of programs and services on However, conscientious objectors are exempt from Sec. 17 as long as
reproductive health. their religious beliefs do not allow them to render RH service, pro bono
or otherwise
7. To provide that the poor are to be given priority in the
government’s RH program is not a violation of the equal PROCEDURAL
protection clause. In fact, it is pursuant to Section 11, Article
1. In this case, the Court is of the view that an actual case or
XIII of the Constitution, which states that the State shall
controversy exists and that the same is ripe for judicial
prioritize the needs of the underprivileged, sick elderly,
determination. Considering that the RH Law and its
disabled, women, and children and that it shall endeavor to
implementing rules have already taken effect and that
provide medical care to paupers.
budgetary measures to carry out the law have already been
The RH Law does not only seek to target the poor to reduce their passed, it is evident that the subject petitions present a
number, since Section 7 of the RH Law prioritizes poor and justiciable controversy. As stated earlier, when an action of
marginalized couples who are suffering from fertility issues and desire the legislative branch is seriously alleged to have infringed the
to have children. In addition, the RH Law does not prescribe the Constitution, it not only becomes a right, but also a duty of the
number of children a couple may have and does not impose conditions Judiciary to settle the dispute.
upon couples who intend to have children. The RH Law only seeks to
provide priority to the poor. Moreover, the petitioners have shown that the case is so because
medical practitioners or medical providers are in danger of being
The exclusion of private educational institutions from the mandatory criminally prosecuted under the RH Law for vague violations thereof,
RH education program under Section 14 is valid. There is a need to particularly public health officers who are threatened to be dismissed
recognize the academic freedom of private educational institutions from the service with forfeiture of retirement and other benefits. They
especially with respect to religious instruction and to consider their must, at least, be heard on the matter now.
sensitivity towards the teaching of reproductive health education
2. In this jurisdiction, the application of doctrines originating from
8. The requirement under Sec. 17 of the RH Law for private and the U.S. has been generally maintained, albeit with some
non-government health care service providers to render 48 modifications. While the Court has withheld the application of
hours of pro bonoRH services does not amount to involuntary facial challenges to strictly penal statues, it has expanded its
servitude, for two reasons. First, the practice of medicine is scope to cover statutes not only regulating free speech, but
undeniably imbued with public interest that it is both the power also those involving religious freedom, and other fundamental
and a duty of the State to control and regulate it in order to rights. The underlying reason for this modification is simple.
protect and promote the public welfare. Second, Section 17 For unlike its counterpart in the U.S., this Court, under its
only encourages private and non-government RH service expanded jurisdiction, is mandated by the Fundamental Law
not only to settle actual controversies involving rights which

29 | P a g e
CONSOLIDATED CASE DIGESTS in Political Law Review (1st Batch)

are legally demandable and enforceable, but also to the issues raised herein have potentially pervasive influence on the
determine whether or not there has been a grave abuse of social and moral well being of this nation, specially the youth; hence,
discretion amounting to lack or excess of jurisdiction on the their proper and just determination is an imperative need. This is in
part of any branch or instrumentality of the Government. accordance with the well-entrenched principle that rules of procedure
Verily, the framers of Our Constitution envisioned a proactive are not inflexible tools designed to hinder or delay, but to facilitate and
Judiciary, ever vigilant with its duty to maintain the supremacy promote the administration of justice. Their strict and rigid application,
which would result in technicalities that tend to frustrate, rather than
of the Constitution.
promote substantial justice, must always be eschewed.
Consequently, considering that the foregoing petitions have seriously 4. Most of the petitions are praying for injunctive reliefs and so the
alleged that the constitutional human rights to life, speech and religion Court would just consider them as petitions for prohibition under Rule
and other fundamental rights mentioned above have been violated by 65, over which it has original jurisdiction. Where the case has far-
the assailed legislation, the Court has authority to take cognizance of reaching implications and prays for injunctive reliefs, the Court may
these kindred petitions and to determine if the RH Law can indeed consider them as petitions for prohibition under Rule 65.
pass constitutional scrutiny. To dismiss these petitions on the simple
expedient that there exist no actual case or controversy, would 5. The RH Law does not violate the one subject/one bill rule. In this
diminish this Court as a reactive branch of government, acting only case, a textual analysis of the various provisions of the law shows that
when the Fundamental Law has been transgressed, to the detriment both “reproductive health” and “responsible parenthood” are
of the Filipino people. interrelated and germane to the overriding objective to control the
population growth. As expressed in the first paragraph of Section 2 of
3. Even if the constitutionality of the RH Law may not be assailed the RH Law:
through an “as-applied challenge, still, the Court has time and
again acted liberally on the locus standi requirement. It has SEC. 2. Declaration of Policy. – The State recognizes and guarantees
accorded certain individuals standing to sue, not otherwise the human rights of all persons including their right to equality and
directly injured or with material interest affected by a nondiscrimination of these rights, the right to sustainable human
Government act, provided a constitutional issue of development, the right to health which includes reproductive health,
transcendental importance is invoked. The rule on locus the right to education and information, and the right to choose and
make decisions for themselves in accordance with their religious
standi is, after all, a procedural technicality which the Court
convictions, ethics, cultural beliefs, and the demands of responsible
has, on more than one occasion, waived or relaxed, thus
parenthood.
allowing non-traditional plaintiffs, such as concerned citizens,
taxpayers, voters or legislators, to sue in the public interest, Considering the close intimacy between “reproductive health” and
albeit they may not have been directly injured by the operation “responsible parenthood” which bears to the attainment of the goal of
of a law or any other government act. achieving “sustainable human development” as stated under its terms,
the Court finds no reason to believe that Congress intentionally sought
The present action cannot be properly treated as a petition for to deceive the public as to the contents of the assailed legislation.
prohibition, the transcendental importance of the issues involved in
this case warrants that the Court set aside the technical defects and
take primary jurisdiction over the petition at bar. One cannot deny that

30 | P a g e
CONSOLIDATED CASE DIGESTS in Political Law Review (1st Batch)

Accordingly, the Court declares R.A. No. 10354 as NOT do any act that hinders the full implementation of a reproductive health
UNCONSTITUTIONAL except with respect to the following provisions program, regardless of his or her religious beliefs;
which are declared UNCONSTITUTIONAL:
7) Section 17 and the corresponding prov1s10n in the RH-IRR
1) Section 7 and the corresponding provision in the RH-IRR insofar as regarding the rendering of pro bona reproductive health service in so
they: a) require private health facilities and non-maternity specialty far as they affect the conscientious objector in securing PhilHealth
hospitals and hospitals owned and operated by a religious group to accreditation; and
refer patients, not in an emergency or life-threatening case, as defined
under Republic Act No. 8344, to another health facility which is 8) Section 3.0l(a) and Section 3.01 G) of the RH-IRR, which added the
conveniently accessible; and b) allow minor-parents or minors who qualifier “primarily” in defining abortifacients and contraceptives, as
have suffered a miscarriage access to modem methods of family they are ultra vires and, therefore, null and void for contravening
planning without written consent from their parents or guardian/s; Section 4(a) of the RH Law and violating Section 12, Article II of the
Constitution.
2) Section 23(a)(l) and the corresponding provision in the RH-IRR,
particularly Section 5 .24 thereof, insofar as they punish any 19.) LOZANO vs. NOGRALES
healthcare service provider who fails and or refuses to disseminate FACTS: The two petitions, filed by their respective petitioners in their
information regarding programs and services on reproductive health capacities as concerned citizens and taxpayers, prayed for the
regardless of his or her religious beliefs. nullification of House Resolution No. 1109 entitled “A Resolution
3) Section 23(a)(2)(i) and the corresponding provision in the RH-IRR Calling upon the Members of Congress to Convene for the Purpose of
insofar as they allow a married individual, not in an emergency or life- Considering Proposals to Amend or Revise the Constitution, Upon a
threatening case, as defined under Republic Act No. 8344, to undergo Three-fourths Vote of All the Members of Congress,” convening the
reproductive health procedures without the consent of the spouse; Congress into a Constituent Assembly to amend the 1987
Constitution. In essence, both petitions seek to trigger a justiciable
4) Section 23(a)(2)(ii) and the corresponding provision in the RH-IRR controversy that would warrant a definitive interpretation by this Court
insofar as they limit the requirement of parental consent only to of Section 1, Article XVII, which provides for the procedure for
elective surgical procedures. amending or revising the Constitution.

5) Section 23(a)(3) and the corresponding provision in the RH-IRR, The petitioners contend that the House Resolution contradicts the
particularly Section 5.24 thereof, insofar as they punish any healthcare procedures set forth by the 1987 Constitution regarding the
service provider who fails and/or refuses to refer a patient not in an amendment or revision of the same as the separate voting of the
emergency or life-threatening case, as defined under Republic Act No. members of each House (the Senate and the House of
8344, to another health care service provider within the same facility Representatives) is deleted and substituted with a vote of three-
or one which is conveniently accessible regardless of his or her fourths of all the Members of Congress (i.e., ¾ of the “members of
religious beliefs; Congress” without distinction as to which institution of Congress they
belong to).
6) Section 23(b) and the corresponding provision in the RH-IRR,
particularly Section 5 .24 thereof, insofar as they punish any public ISSUE: Whether the court has the power to review the case of the
officer who refuses to support reproductive health programs or shall validity of House Resolution No. 1109.

31 | P a g e
CONSOLIDATED CASE DIGESTS in Political Law Review (1st Batch)

HELD: No. The Supreme Court cannot indulge petitioners’ procedure have yet been adopted. More importantly, no proposal has
supplications. While some may interpret petitioners’ moves as yet been made, and hence, no usurpation of power or gross abuse of
vigilance in preserving the rule of law, a careful perusal of their discretion has yet taken place. In short, House Resolution No. 1109
petitions would reveal that they cannot hurdle the bar of justiciability involves a quintessential example of an uncertain contingent future
set by the Court before it will assume jurisdiction over cases involving event that may not occur as anticipated, or indeed may not occur at
constitutional disputes. all. The House has not yet performed a positive act that would warrant
an intervention from this Court.
The Court’s power of review may be awesome, but it is limited to
actual cases and controversies dealing with parties having adversely As in the case of Tan v. Macapagal, as long as any proposed
legal claims, to be exercised after full opportunity of argument by the amendment is still unacted on by it, there is no room for the
parties, and limited further to the constitutional question raised or the interposition of judicial oversight. Only after it has made concrete what
very lis mota presented. The “case-or-controversy” requirement bans it intends to submit for ratification may the appropriate case be
this court from deciding “abstract, hypothetical or contingent instituted. Until then, the courts are devoid of jurisdiction
questions,” lest the court give opinions in the nature of advice
concerning legislative or executive action A party will be allowed to litigate only when he can demonstrate that
(1) he has personally suffered some actual or threatened injury
An aspect of the “case-or-controversy” requirement is the requisite of because of the allegedly illegal conduct of the government; (2) the
“ripeness.” In the United States, courts are centrally concerned with injury is fairly traceable to the challenged action; and (3) the injury is
whether a case involves uncertain contingent future events that may likely to be redressed by the remedy being sought. In the cases at bar,
not occur as anticipated, or indeed may not occur at all. Another petitioners have not shown the elemental injury in fact that would
approach is the evaluation of the twofold aspect of ripeness: first, the endow them with the standing to sue. Locus standi requires a personal
fitness of the issues for judicial decision; and second, the hardship to stake in the outcome of a controversy for significant reasons. It
the parties entailed by withholding court consideration. In our assures adverseness and sharpens the presentation of issues for the
jurisdiction, the issue of ripeness is generally treated in terms of actual illumination of the Court in resolving difficult constitutional questions.
injury to the plaintiff. Hence, a question is ripe for adjudication when The lack of petitioners’ personal stake in this case is no more evident
the act being challenged has had a direct adverse effect on the than in Lozano’s three-page petition that is devoid of any legal or
individual challenging it. An alternative road to review similarly taken jurisprudential basis.
would be to determine whether an action has already been
accomplished or performed by a branch of government before the Neither can the lack of locus standi be cured by the claim of petitioners
courts may step in. that they are instituting the cases at bar as taxpayers and concerned
citizens. A taxpayer’s suit requires that the act complained of directly
In the present case, the fitness of petitioners’ case for the exercise of involves the illegal disbursement of public funds derived from taxation.
judicial review is grossly lacking. In the first place, petitioners have not It is undisputed that there has been no allocation or disbursement of
sufficiently proven any adverse injury or hardship from the act public funds in this case as of yet.
complained of. In the second place, House Resolution No. 1109 only
resolved that the House of Representatives shall convene at a future The possible consequence of House Resolution No. 1109 is yet
time for the purpose of proposing amendments or revisions to the unrealized and does not infuse petitioners with locus standi
Constitution. No actual convention has yet transpired and no rules of

32 | P a g e
CONSOLIDATED CASE DIGESTS in Political Law Review (1st Batch)

The rule on locus standi is not a plain procedural rule but a Petitioner filed his answer and counter-protested 560 precincts
constitutional requirement derived from Section 1, Article VIII of the claiming that massive fraud through deliberate misreading,
Constitution, which mandates courts of justice to settle only “actual miscounting and misappreciation of ballots were also committed
controversies involving rights which are legally demandable and against him in said precincts resulting in the reduction of his votes in
enforceable.” order to favor private respondent.
Moreover, while the Court has taken an increasingly liberal approach
to the rule of locus standi, evolving from the stringent requirements of The HRET then ordered that all ballot boxes and other election
“personal injury” to the broader “transcendental importance” doctrine, materials involved in the protest and counter-protest be collected and
such liberality is not to be abused. It is not an open invitation for the retrieved, and brought to its offices for custody.
ignorant and the ignoble to file petitions that prove nothing but their
cerebral deficit. In the preliminary conference petitioner and private respondent
agreed that, since the total number of the protested precincts was less
IN VIEW WHEREOF, the petitions are dismissed. than 50% of the total number of the precincts in the 2 nd legislative
20.) DUENAS vs. HRET district of Taguig City, all of the protested precincts would be revised
without need of designation of pilot precincts by private respondent
Sed quis custodiet ipsos custodies? (But who is pursuant to Rule 88 of the HRET Rules.
to guard the guardians themselves?)
The HRET thereafter directed the revision of ballots. Reception of
FACTS: evidence of the contending parties followed after the revision of ballots
in 100% of the protested precincts and 25% pilot of the counter-
Petitioner Henry Jun Dueas, Jr. and private respondent Angelito Jett protested precincts. The case was then submitted for resolution upon
P. Reyes were rival candidates for the position of congressman in the submission by the parties of their memoranda.
2nd legislative district of Taguig City in the May 14, 2007 synchronized
national and local elections. After the canvass of the votes, petitioner The HRET directed the continuation of the revision and appreciation
was proclaimed the winner, having garnered 28,564 votes as opposed of the remaining 75% of the counter-protested precincts pursuant to
to private respondents 27,107 votes. Rule 88 of the HRET Rules, [i]t appearing that the [HRET] cannot
determine the true will of the electorate from the initial revision and
Private respondent filed an election protest ad cautelam, in the HRET appreciation of the 100% protested precincts and 25% counter-
and prayed for a revision/recount in 170 of the 732 precincts in the protested precincts and in view of the discovery of fake/spurious
2nd legislative district of Taguig City so that the true and real mandate ballots in some of the protested and counter-protested precincts.
of the electorate may be ascertained. In support of his protest, he
alleged that he was cheated in the protested precincts through In another order, HRET directed petitioner to augment his cash
insidious and well-orchestrated electoral frauds and anomalies which deposit in the amount of P320, 000 to cover the expenses of the
resulted in the systematic reduction of his votes and the corresponding revision of ballots in the remaining 75% counter-protested precincts
increase in petitioner’s votes. within a non-extendible period of ten days from notice.

33 | P a g e
CONSOLIDATED CASE DIGESTS in Political Law Review (1st Batch)

Instead of complying with the order, petitioner filed an urgent motion RULING:
to withdraw/abandon the remaining 75% counter-protested precincts.
This was denied by the HRET in Resolution No. 08-353 reiterating its
The petition has no merit.
order directing the continuation of the revision of ballots in the
remaining 75% counter-protested precincts and recalling its order
requiring petitioner to augment his cash deposit. The Tribunal instead The base of the decision is not only on the constitutional authority of
ordered the use of its own funds for the revision of the remaining 75% the HRET as the sole judge of all contests relating to the election,
counter-protested precincts. returns and qualifications of its members but also on the limitation of
the Courts power of judicial review.
In issuing Resolution No. 08-353, the HRET invoked Rule 88 of the
HRET Rules and settled jurisprudence, ruling that it had the discretion The Court itself has delineated the parameters of its power of review
either to dismiss the protest or counter-protest, or to continue with the in cases involving the HRET
revision if necessitated by reasonable and sufficient grounds affecting
the validity of the election. This was with the end in view of ... so long as the Constitution grants the HRET the
ascertaining the true choice of the electorate. It was the HRET’s power to be the sole judge of all contests relating to
position that the mere filing of a motion to withdraw/abandon the the election, returns and qualifications of members of
unrevised precincts did not automatically divest the HRET of its the House of Representatives, any final action taken
jurisdiction over the same. Moreover, it ruled that its task of by the HRET on a matter within its jurisdiction shall,
determining the true will of the electorate was not confined to the as a rule, not be reviewed by this Court . the power
examination of contested ballots. Under its plenary power, it granted to the Electoral Tribunal x x x excludes
could motu propio review the validity of every ballot involved in a the exercise of any authority on the part of this
protest or counter-protest and the same could not be frustrated by the Court that would in any wise restrict it or curtail it
mere expedient of filing a motion to withdraw/abandon the remaining or even affect the same.
counter-protested precincts. Convinced that it could not determine the
true will of the electorate of the 2nd legislative district of Taguig City on Guided by this basic principle, the Court will neither assume a power
the basis alone of the initial revision of the 100% protested precincts that belongs exclusively to the HRET nor substitute its own judgment
and the 25% counter-protested precincts, it had no other recourse but for that of the Tribunal.
to continue the revision and appreciation of all the remaining 75%
counter-protested precincts.
The acts complained of in this case pertain to the HRET’s exercise of
its discretion, an exercise which was well within the bounds of its
Aggrieved by the HRETs Resolution No. 08-353 dated November 27, authority.
2008, petitioner elevated the matter to the Supreme Court.
Indeed, due regard and respect for the authority of the HRET as an
ISSUE: Whether or not the Court has jurisdiction to hear and independent constitutional body require that any finding of grave
decide cases over election protests heard and decided by the abuse of discretion against that body should be based on firm and
HRET. convincing proof, not on shaky assumptions. Any accusation of grave

34 | P a g e
CONSOLIDATED CASE DIGESTS in Political Law Review (1st Batch)

abuse of discretion on the part of the HRET must be established government infrastructure projects with available funding are about to
by a clear showing of arbitrariness and improvidence. But the Court be implemented.
finds no evidence of such grave abuse of discretion by the HRET.
Petitioners directly filed a petition for prohibition and mandamus
In Co v. HRET, the Court held that: before the Court, seeking to compel the Secretary of Interior and Local
Government, et al. To first secure an eviction and/or demolition order
The Court does not venture into the perilous area from the court prior to their implementation of Section 28 (a) and (b)
of trying to correct perceived errors of of RA 7279.
independent branches of the Government. It The petitioners justify their direct recourse before this Court by
comes in only when it has to vindicate a denial of due generally averring that they have no plain, speedy and adequate
process or correct an abuse of discretion so grave or remedy in the ordinary course of law.4 They also posit that the
glaring that no less than the Constitution calls for respondents gravely abused their discretion in implementing Section
remedial action. 28 (a) and (b) of RA 7279 which are patently unconstitutional.

The Constitution mandates that the HRET shall be the sole judge of The petitioners argue that Section 28 (a) and (b) of RA 7279 offend
all contests relating to the election, returns and qualifications of its their constitutional right to due process because they warrant evictions
members. By employing the word sole, the Constitution is emphatic and demolitions without any court order. They point out that Section
that the jurisdiction of the HRET in the adjudication of election contests 6, Article 3 of the 1987 Constitution expressly prohibits the impairment
involving its members is exclusive and exhaustive. Its exercise of of liberty of abode unless there is a court order.
power is intended to be its own full, complete and unimpaired.
The Mayor of Navotas argues that Section 10, Article 13 of the 1987
Constitution allows evictions and demolitions to be conducted even
21.) KALIPUNAN NG DAMAY ANG MAHIBIRAP, INC vs. without a court order provided they are done in accordance with the
JESSIE ROBREDO, in his capacity as Secretary, Department law and in a just and humane manner. The Mayor of San Juan
of Interior and Local Government. similarly argues petitioners’ premature invocation of the Court’s power
of judicial review and their violation of the principle of hierarchy of
FACTS: The members of petitioners Kalipunan ng Damayang
courts are fatal to their cause of action. Moreover, the petitioners failed
Mahihirap, Inc. were/are occupying parcels of land owned by and
to substantiate the material allegations in the petition. He additionally
located in the cities of San Juan, Navotas and Quezon. These LGUs
argues that his faithful implementation of RA 7279, which the
sent the petitioners notices of eviction and demolition pursuant to
legislature enacted in the exercise of police power, does not amount
Section 28 (a) and (b) of RA 7279 in order to give way to the
to grave abuse of discretion
implementation and construction of infrastructure projects2 in the
areas illegally occupied by the petitioners. The Secretary of Interior and Local Government and the National
Housing Authority (NHA) General Manager adopt the Mayor of
RA 7279 authorize evictions and demolitions without any court order
Navotas’ position that the petition is procedurally infirm. They further
when: (1) persons or entities occupy danger areas such as esteros,
argue that the liberty of abode is not illimitable and does not include
railroad tracks, garbage dumps, riverbanks, shorelines, waterways,
the right to encroach upon other person properties. They also reiterate
and other public places suchas sidewalks, roads, parks, and
that Section 28 of RA 7279 provides sufficient safeguards in ensuring
playgrounds; and (2) persons or entities occupy areas where

35 | P a g e
CONSOLIDATED CASE DIGESTS in Political Law Review (1st Batch)

that evictions and demolitions are carried out in a just and humane of a law, there must be a clear and unequivocal breach of the
manner. Constitution, and not one that is doubtful, speculative or
argumentative.23
ISSUE: Whether Section 28 (a) and (b) of RA 7279 are violative of
Sections 1 and 6, Article 3 of the 1987 Constitution. We carefully read the petitions and we conclude that they fail to
compellingly show the necessity of examining the constitutionality of
HELD: NO. Even if we treat the present petition as one for certiorari Section 28 (a) and (b) of RA 7279 in the light of Sections 1 and 6,
since it assails the constitutionality of Section 28 (a) and (b) of RA Article 3 of the 1987 Constitution. We further stated that demolitions
7279, the petition must necessarily fail for failure to show the essential and evictions may be validly carried out even without a judicial order.
requisites that would warrant the Court’s exercise of judicial review. It
is a rule firmly entrenched in our jurisprudence that the courts will not 22.) ROMUALDEZ vs. COMELEC
determine the constitutionality of a law unless the following requisites
are present: (1) the existence of an actual case or controversy FACTS: Private respondent Dennis Garay, along with Angelino
involving a conflict of legal rights susceptible of judicial determination; Apostol filed a Complaint-Affidavit with the COMELEC thru the Office
(2) the existence of personal and substantial interest on the part of the of the Election Officer in Burauen, Leyte, charging petitioners with
party raising the constitutional question; (3) recourse to judicial review violation of Section 261(y)(2 and Section 261(y)(5) of the Omnibus
is made at the earliest opportunity; and (4) the resolution of the Election Code, similarly referred to as Batas Pambansa Blg. 881; and
constitutional question must be necessary to the decision of the case Section 12 of Republic Act No. 8189.

Save for the petition pertaining to the City of Quezon’s threat of Private respondent deposed, inter alia, that: petitioners are of legal
eviction and demolition, this case no longer presents a justiciable ages and residents of 113 Mariposa Loop, Mariposa Street, Bagong
controversy with respect to the Mayors of Navotas and San Juan. We Lipunan ng Crame, Quezon City; on 9 May 2000 and 11 May 2000,
take note of the Comments of these Mayors who alleged that they had petitioners Carlos S. Romualdez and Erlinda R. Romualdez, applied
already successfully evicted the concerned petitioners in their for registration as new voters with the Office of the Election Officer of
respective cities at the time of the filing of the petition. Burauen, Leyte, as evidenced by Voter Registration Record Nos.
42454095 and 07902952, respectively; in their sworn applications,
What further constrains this Court from touching on the issue of petitioners made false and untruthful representations in violation of
constitutionality is the fact that this issue is not the lis mota of this case. Section 10 of Republic Act Nos. 8189, by indicating therein that they
Lis mota literally means "the cause of the suit or action"; it is rooted in are residents of 935 San Jose Street, Burauen, Leyte, when in truth
the principle of separation of powers and is thus merely an offshoot of and in fact, they were and still are residents of 113 Mariposa Loop,
the presumption of validity accorded the executive and legislative acts Mariposa Street, Bagong Lipunan ng Crame, Quezon City, and
of our coequal branches of the government. registered voters of Barangay Bagong Lipunan ng Crame, District IV,
Quezon City, Precinct No. 4419-A, as evidenced by Voter Registration
This means that the petitioner who claims the unconstitutionality of a Record Nos. 26195824 and 26195823; and that petitioners, knowing
law has the burden of showing first that the case cannot be resolved fully well said truth, intentionally and willfully, did not fill the blank
unless the disposition of the constitutional question that he raised is spaces in said applications corresponding to the length of time which
unavoidable. If there is some other ground upon which the court may they have resided in Burauen, Leyte. In fine, private respondent
rest its judgment, that course will be adopted and the question of charged petitioners, to wit:
constitutionality should be avoided.22 Thus, to justify the nullification

36 | P a g e
CONSOLIDATED CASE DIGESTS in Political Law Review (1st Batch)

Respondent-spouses, Carlos Sison Romualdez and Erlinda Reyes On 12 January 2006, Alioden D. Dalaig, Director IV, Law Department
Romualdez committed and consummated election offenses in of the COMELEC filed with the RTC, Burauen, Leyte, separate
violation of our election laws, specifically, Sec. 261, paragraph (y), Informations against petitioner Carlos S. Romualdez for violation of
subparagraph (2), for knowingly making any false or untruthful Section 10(g), in relation to Section 45(j) of Republic Act No. 8189,
statements relative to any data or information required in the and against petitioner Erlinda R. Romualdez for violation of Section
application for registration, and of Sec. 261, paragraph (y), 10(g), in relation to Section 45(j) of Republic Act No. 8189,
subparagraph (5), committed by any person who, being a registered subsequently docketed as Crim. Case No. BN-06-03-4185 and Crim.
voter, registers anew without filing an application for cancellation of Case No. BN-06-03-4183, respectively. Moreover, separate
his previous registration, both of the Omnibus Election Code (BP Blg. Informations for violation of Section 10(j), in relation to Section 45(j) of
881), and of Sec. 12, RA 8189 (Voter Registration Act) for failure to Republic Act No. 8189 were filed against petitioners.
apply for transfer of registration records due to change of residence to
another city or municipality." ISSUE: W/N RESPONDENT COMMISSION ON ELECTIONS
GRAVELY ABUSED ITS DISCRETION AMOUNTING TO LACK OF
On 28 November 2003, Atty. Maria Norina S. Tangaro-Casingal, OR IN EXCESS OF ITS JURISDICTION
COMELEC Investigating Officer, issued a Resolution, recommending
to the COMELEC Law Department (Investigation and Prosecution RULING: NO. The constitutional grant of prosecutorial power in the
Division), the filing of the appropriate Information against petitioners, COMELEC finds statutory expression under Section 265 of Batas
disposing, thus: Pambansa Blg. 881, otherwise known as the Omnibus Election
Code. The task of the COMELEC whenever any election offense
PREMISES CONSIDERED, the Law Department (Investigation and charge is filed before it is to conduct the preliminary investigation of
Prosecution Division), RECOMMENDS to file the necessary the case, and make a determination of probable cause. Under Section
information against Carlos Sison Romualdez before the proper 8(b), Rule 34 of the COMELEC Rules of Procedure, the investigating
Regional Trial Court for violation of Section 10 (g) and (j) in relation to officer makes a determination of whether there is a reasonable ground
Section 45 (j) of Republic Act 8189 and to authorize the Director IV of to believe that a crime has been committed. In Baytan v.
the Law Department to designate a Comelec Prosecutor to handle the COMELEC, this Court, sufficiently elucidated on the matter of
prosecution of the case with the duty to submit periodic report after probable cause in the prosecution of election offenses, viz:
every hearing of the case.
It is also well-settled that the finding of probable cause in the
On 11 June 2004, the COMELEC En Banc found no reason to depart prosecution of election offenses rests in the COMELEC's sound
from the recommendatory Resolution of 28 November 2003, and discretion. The COMELEC exercises the constitutional authority to
ordered, viz: investigate and, where appropriate, prosecute cases for violation of
election laws, including acts or omissions constituting election frauds,
WHEREFORE, premises considered, the Law Department is hereby offense and malpractices. Generally, the Court will not interfere with
directed to file the appropriate information with the proper court such finding of the COMELEC absent a clear showing of grave abuse
against respondents CARLOS S. ROMUALDEZ AND ERLINDA of discretion. This principle emanates from the COMELEC's exclusive
ROMUALDEZ for violation of Section 10 (g) and (j) in relation to power to conduct preliminary investigation of all election offenses
Section 45 (j) of the Republic Act No. 8189. punishable under the election laws and to prosecute the same, except
as may otherwise be provided by law.

37 | P a g e
CONSOLIDATED CASE DIGESTS in Political Law Review (1st Batch)

It is succinct that courts will not substitute the finding of be doubted. For the business of a court of justice is to be an impartial
probable cause by the COMELEC in the absence of grave abuse tribunal, and not to get involved with the success or failure of the
of discretion. The abuse of discretion must be so patent and prosecution to prosecute. Every now and then, the prosecution may
gross as to amount to an evasion of a positive duty or a virtual err in the selection of its strategies, but such errors are not for neutral
refusal to perform a duty enjoined by law, or to act at all in courts to rectify, any more than courts should correct the blunders of
contemplation of law as where the power is exercised in an the defense.
arbitrary and despotic manner by reason of passion or hostility.
23.) DAVID vs. MACAPAGAL-ARROYO
According to the COMELEC En Banc, the investigating officer, in the
case at bar, held that there was sufficient cause for the filing of criminal G.R. No. 171396 May 3, 2006
charges against petitioners, and found no reason to depart therefrom. FACTS: On February 24, 2006, as the nation celebrated the 20th
Without question, on May 9 and 11 of 2001, petitioners applied for Anniversary of the Edsa People Power I, President Arroyo issued PP
registration as new voters with the Office of the Election Officer of 1017 declaring a state of national emergency, thus:
Burauen, Leyte, notwithstanding the existence of petitioners’
registration records as registered voters of Precinct No. 4419-A of NOW, THEREFORE, I, Gloria Macapagal-Arroyo, President of the
Barangay Bagong Lipunan ng Crame, District IV, Quezon City. The Republic of the Philippines and Commander-in-Chief of the Armed
directive by the COMELEC which affirmed the Resolution of 28 Forces of the Philippines, by virtue of the powers vested upon me by
November 2000 of Investigating Officer Atty. Tangaro-Casingal does Section 18, Article 7 of the Philippine Constitution which states that:
not appear to be wanting in factual basis, such that a reasonably “The President. . . whenever it becomes necessary, . . . may call out
prudent man would conclude that there exists probable cause to hold (the) armed forces to prevent or suppress. . .rebellion. . .,” and in my
petitioners for trial. capacity as their Commander-in-Chief, do hereby command the
Armed Forces of the Philippines, to maintain law and order throughout
That the Constitution grants to the COMELEC the power to the Philippines, prevent or suppress all forms of lawless violence as
prosecute cases or violations of election laws. Article IX (C), well as any act of insurrection or rebellion and to enforce obedience
Section 2 (6) of the 1987 Constitution, provides: to all the laws and to all decrees, orders and regulations promulgated
(6) File, upon a verified complaint, or on its own initiative, by me personally or upon my direction; and as provided in Section 17,
petitions in court for inclusion or exclusion of voters; investigate Article 12 of the Constitution do hereby declare a State of National
and where appropriate, prosecute cases or violations of election Emergency.
laws, including acts or omissions constituting election frauds, On the same day, the President issued G. O. No. 5 implementing PP
offenses, and malpractices. 1017. Respondents stated that the proximate cause behind the
This power to prosecute necessarily involves the power to determine executive issuances was the conspiracy among some military officers,
who shall be prosecuted, and the corollary right to decide whom not leftist insurgents of the New People’s Army (NPA), and some
to prosecute. Evidently, must this power to prosecute also include the members of the political opposition in a plot to unseat or assassinate
right to determine under which laws prosecution will be pursued. The President Arroyo. They considered the aim to oust or assassinate the
courts cannot dictate the prosecution nor usurp its discretionary President and take-over the reigns of government as a clear and
powers. As a rule, courts cannot interfere with the prosecutor’s present danger.
discretion and control of the criminal prosecution. Its rationale cannot

38 | P a g e
CONSOLIDATED CASE DIGESTS in Political Law Review (1st Batch)

ISSUE: Whether the issuance of PP 1021 renders the petitions moot violation of the Constitution; second, the exceptional character of the
and academic situation and the paramount public interest is involved; third, when
constitutional issue raised requires formulation of controlling principles
HELD: NO. The power of judicial review may be exercised only when to guide the bench, the bar, and the public; and fourth, the case is
the following requisites are present: first, there must be an actual case capable of repetition yet evading review.
or controversy; second, petitioners have to raise a question of
constitutionality; third, the constitutional question must be raised at the All the foregoing exceptions are present here and justify the Supreme
earliest opportunity; and fourth, the decision of the constitutional Court’s assumption of jurisdiction over the instant petitions. Petitioners
question must be necessary to the determination of the case itself. alleged that the issuance of PP 1017 and G.O. No. 5 violates the
Constitution. There is no question that the issues being raised affect
An actual case or controversy involves a conflict of legal right, an the public’s interest, involving as they do the people’s basic rights to
opposite legal claims susceptible of judicial resolution. It is “definite freedom of expression, of assembly and of the press. Moreover, the
and concrete, touching the legal relations of parties having adverse Court has the duty to formulate guiding and controlling constitutional
legal interest;” a real and substantial controversy admitting of specific precepts, doctrines or rules. It has the symbolic function of educating
relief. The Solicitor General refutes the existence of such actual case the bench and the bar, and in the present petitions, the military and
or controversy, contending that the present petitions were rendered the police, on the extent of the protection given by constitutional
“moot and academic” by President Arroyo’s issuance of PP 1021. guarantees. And lastly, respondents’ contested actions are capable of
Such contention lacks merit. repetition. Certainly, the petitions are subject to judicial review.

A moot and academic case is one that ceases to present a justiciable ISSUE: Whether or not the Supreme Court may review the factual
controversy by virtue of supervening events, so that a declaration bases of the President’s exercise of his Commander-in-Chief power
thereon would be of no practical use or value. Generally, courts HELD: YES. In IBP v. Zamora, while the Court considered the
decline jurisdiction over such case or dismiss it on ground of President’s “calling-out” power as a discretionary power solely vested
mootness. in his wisdom, it stressed that this does not prevent an examination of
The Court holds that President Arroyo’s issuance of PP 1021 did not whether such power was exercised within permissible constitutional
render the present petitions moot and academic. During the eight (8) limits or whether it was exercised in a manner constituting grave abuse
days that PP 1017 was operative, the police officers, according to of discretion. This ruling is mainly a result of the Court’s reliance on
petitioners, committed illegal acts in implementing it. Are PP 1017 and Section 1, Article VIII of 1987 Constitution which fortifies the authority
G.O. No. 5 constitutional or valid? Do they justify these alleged illegal of the courts to determine in an appropriate action the validity of the
acts? These are the vital issues that must be resolved in the present acts of the political departments. Under the new definition of judicial
petitions. It must be stressed that “an unconstitutional act is not a law, power, the courts are authorized not only “to settle actual
it confers no rights, it imposes no duties, it affords no protection; it is controversies involving rights which are legally demandable and
in legal contemplation, inoperative.” enforceable,” but also “to determine whether or not there has been a
grave abuse of discretion amounting to lack or excess of jurisdiction
The “moot and academic” principle is not a magical formula that can on the part of any branch or instrumentality of the government.” The
automatically dissuade the courts in resolving a case. Courts will latter part of the authority represents a broadening of judicial power to
decide cases, otherwise moot and academic, if: first, there is a grave enable the courts of justice to review what was before a forbidden

39 | P a g e
CONSOLIDATED CASE DIGESTS in Political Law Review (1st Batch)

territory, to wit, the discretion of the political departments of the elections. The disqualification of the ABC Party-List resulted in the re-
government. It speaks of judicial prerogative not only in terms computation of the party-list allocations in the House of
of power but also of duty. Representatives, in which the COMELEC followed the formula
outlined in the case of Barangay Association for National
As to how the Court may inquire into the President’s exercise of Advancement and Transparency (BANAT) v. Commission on
power, Lansang v. Garcia adopted the test that judicial inquiry can go Elections.1
no further than to satisfy the Court not that the President’s decision
is correct, but that the President did not act arbitrarily. Thus, the The COMELEC then issued Minute Resolution No. 12-0859, in which
standard laid down is not correctness, but arbitrariness. In Integrated it resolved: to grant the proclamation of (Alay Buhay) Party-List;
Bar of the Philippines, the Court further ruled that it is incumbent upon denied the motion of (Senior Citizens) Party-List; and confirmed the
the petitioner to show that the President’s decision is totally bereft of re-computation of the allocations of seats which resulted in (Alay
factual basis and that if he fails, by way of proof, to support his Buhay) Party-List as a winning party-list group in the Party-List System
assertion, then the Supreme Court cannot undertake an independent of Representation in the House of Representatives in the May 10,
investigation beyond the pleadings. 2010 Automated National and Local Election. It also declared the First
(1st) NOMINEE of Alay Buhay Community Development Foundation,
Petitioners failed to show that President Arroyo’s exercise of the Inc. (Alay Buhay) Party-List, as the FIRST (1st) SITTING
calling-out power, by issuing PP 1017, is totally bereft of factual basis. REPRESENTATIVE in the Party-List System.
A reading of the Solicitor General’s Consolidated Comment and
Memorandum shows a detailed narration of the events leading to the On 25 October 2012, petitioners Association of Flood Victims and
issuance of PP 1017, with supporting reports forming part of the Jaime Aguilar Hernandez (Hernandez) filed with this Court a special
records. Mentioned are the escape of the Magdalo Group, their civil action for certiorari and/or mandamus under Rule 65 of the Rules
audacious threat of the Magdalo D-Day, the defections in the military, of Court. Petitioners assert that the COMELEC committed grave
particularly in the Philippine Marines, and the reproving statements abuse of discretion when it issued Minute Resolution No. 12-0859.
from the communist leaders. There was also the Minutes of the Furthermore, petitioners pray for the issuance of a writ of mandamus
Intelligence Report and Security Group of the Philippine Army showing to compel publication of the COMELEC Minute Resolution No. 12-
the growing alliance between the NPA and the military. Petitioners 0859.
presented nothing to refute such events. Thus, absent any contrary
allegations, the Court is convinced that the President was justified in ISSUES:
issuing PP 1017 calling for military aid. (1) Whether the COMELEC committed grave abuse of discretion in
24.) ASSOCIATION OF FLOOD VICTIMS and JAIME AGUILAR issuing Minute Resolution No. 12-0859, and
HERNANDEZ vs. COMMISSION ON ELECTIONS (2) Whether the COMELEC may be compelled through mandamus to
G.R. No. 203775 August 5, 2014 publish Minute Resolution No. 12-0859.

FACTS: On 28 August 2012, the Supreme Court affirmed COMELEC RULING: NO. Petitioners do not have legal capacity to sue. Sections
Resolution SPP 10-013, dated 11 October 2011, cancelling the 1 and 2, Rule 3 of the 1997 Rules of Civil Procedure it provides that,
certificate of registration of the Alliance of Barangay Concerns (ABC) only natural or juridical persons, or entities authorized by law may be
Party-List which won in the party-list elections in the 2010 national

40 | P a g e
CONSOLIDATED CASE DIGESTS in Political Law Review (1st Batch)

parties in a civil action, which must be prosecuted or defended in the Besides, petitioners have no locus standi or legal standing. Locus
name of the real party in interest. standi or legal standing is defined as: x x x a personal and substantial
interest in the case such that the party has sustained or will sustain a
Petitioner Association of Flood Victims is an unincorporated direct injury as a result of the governmental act that is being
association not endowed with a distinct personality of its own. An challenged. The term "interest" means a material interest, an. interest
unincorporated association, in the absence of an enabling law, has no in issue affected by the decree, as distinguished from mere interest in
juridical personality and thus, cannot sue in the name of the the question involved, or a mere incidental interest. The gist of the
association.5 Such unincorporated association is not a legal entity question of standing is whether a party alleges such personal stake in
distinct from its members. If an association, like petitioner Association the outcome of the controversy as to assure that concrete
of Flood Victims, has no juridical personality, then all members of the adverseness which sharpens the presentation of issues upon which
association must be made parties in the civil action.6 In this case, other the court depends for illumination of difficult constitutional questions.12
than his bare allegation that he is the lead convenor of the Association
of Flood Victims, petitioner Hernandez showed no proof that he was In this case, petitioners failed to allege personal or substantial interest
authorized by said association. Aside from petitioner Hernandez, no in the questioned governmental act which is the issuance of
other member was made signatory to the petition. Only petitioner COMELEC Minute Resolution No. 12-0859, which confirmed the re-
Hernandez signed the Verification and Sworn Certification Against computation of the allocation of seats of the Party-List System of
Forum Shopping,7stating that he caused the preparation of the Representation in the House of Representatives in the 10 May 2010
petition. There was no accompanying document showing that the Automated National and Local Elections. Petitioner Association of
other members of the Association of Flood Victims authorized Flood Victims is not even a party-list candidate in the 10 May 2010
petitioner Hernandez to represent them and the association in the elections, and thus, could not have been directly affected by
petition. COMELEC Minute Resolution No. 12-0859.

In Dueñas v. Santos Subdivision Homeowners Association,8 the Court In view of our holding that petitioners do not have legal capacity to sue
held that the Santos Subdivision Homeowners Association (SSHA), and have no standing to file the present petition, we shall no longer
which was an unincorporated association, lacks capacity to sue in its discuss the issues raised in this petition. WHEREFORE, we DISMISS
own name, and that the members of the association cannot represent the petition.
the association without valid authority.
25.) RESIDENT MARINE MAMMALS vs. REYES
More so in this case where there is no showing that petitioner
Hernandez is validly authorized to represent petitioner Association of G.R. No. 180771, April 21, 2015
Flood Victims. Since petitioner Association of Flood Victims has no Justice LEONARDO-DE CASTRO
legal capacity to sue, petitioner Hernandez, who is filing this petition
as a representative of the Association of Flood Victims, is likewise FACTS:
devoid of legal personality to bring an action in court. Neither can  Petitioners Resident Marine Mammals bring their case in their
petitioner Hernandez sue as a taxpayer because he failed to show that personal capacity, alleging that they stand to benefit or be
there was illegal expenditure of money raised by taxation 10 or that injured from the judgment on the issues. The human
public funds are wasted through the enforcement of an invalid or petitioners implead themselves in a representative capacity
unconstitutional law.11 "as legal guardians of the lesser life-forms and as responsible
stewards of God's Creations." They use Oposa v. Factoran,

41 | P a g e
CONSOLIDATED CASE DIGESTS in Political Law Review (1st Batch)

Jr.2 as basis for their claim, asserting their right to enforce simplification of procedures and facilitating court access
international and domestic environmental laws enacted for in environmental cases.” Instead, the Court explained, “the
their benefit under the concept of stipulation pour autrui. As need to give the Resident Marine Mammals legal standing
the representatives of Resident Marine Mammals, the has been eliminated by our Rules, which allow any Filipino
human petitioners assert that they have the obligation to citizen, as a steward of nature, to bring a suit to enforce our
build awareness among the affected residents of Tañon environmental laws.”
Strait as well as to protect the environment, especially in
light of the government's failure, as primary steward, to The Rules provide that parties may only be natural or juridical
do its duty under the doctrine of public trust. persons or entities that may be authorized by statute to be parties in
 Resident Marine Mammals and the human petitioners also a civil action.
assert that through this case, this court will have the
opportunity to lower the threshold for locus standi as an SEC. 2. Parties in interest.-A real party in interest is
exercise of "epistolary jurisdiction." the party who stands to be benefited or injured by the
 The human petitioners want the court to create substantive judgment in the suit, or the party entitled to the avails
and procedural rights for animals through their allegation that of the suit. Unless otherwise authorized by law or
they can speak for them. The Court is asked to accept the these Rules, every action must be prosecuted or
premises that (a) they were chosen by the Resident Marine defended in the name of the real party in interest.
Mammals of Tañon Strait; (b) they were chosen by a (2a)6
representative group of all the species of the Resident Marine
Mammals; (c) they were able to communicate with them; and A litigant who stands to benefit or sustain an injury from
(d) they received clear consent from their animal principals the judgment of a case is a real party in interest. When a case is
that they would wish to use human legal institutions to pursue brought to the courts, the real party in interest must show that
their interests. another party's act or omission has caused a direct injury,
 In the alternative, they want the court to accept through making his or her interest both material and based on an
judicial notice that there is a relationship of guardianship enforceable legal right.
between them and all the resident mammals in the
affected ecology. SEC. 3. Representatives as parties. - Where the
action is allowed to be prosecuted or defended by a
ISSUE/S: Whether marine mammals, through their stewards, have representative or someone acting in a fiduciary
legal standing to pursue the case; capacity, the beneficiary shall be included in the title
of the case and shall be deemed to be the real party
RULING: in interest. A representative may be a trustee of an
express rust, a guardian, an executor or
1. No. As to standing, the Court declined to extend the administrator, or a party authorized by law or these
principle of standing beyond natural and juridical Rules. An agent acting in his own name and for the
persons, even though it recognized that the current trend benefit of an undisclosed principal may sue or be
in Philippine jurisprudence “moves towards

42 | P a g e
CONSOLIDATED CASE DIGESTS in Political Law Review (1st Batch)

sued without joining the principal except when the challenged humans who possess an enforceable
contract involves things belonging to the principal. right but lack the ability to enforce it themselves.”

The rule is two-pronged. First, it defines .a representative as When a court allows guardianship as a basis of
a party who is not bound to directly or actually benefit or suffer from representation, animals are considered as similarly situated as
the judgment, but instead brings a case in favor of an identified real individuals who have enforceable rights but, for a legitimate
party in interest. The representative is an outsider to the cause of reason (e.g., cognitive disability), are unable to bring suit for
action. Second, the rule provides a list of who may be considered as themselves. They are also similar to entities that by their very
"representatives." It is not an exhaustive list, but the rule limits the nature are incapable of speaking for themselves (e.g.,
coverage only to those authorized by law or the Rules of Court. corporations, states, and others).

These requirements should apply even in cases involving the In our jurisdiction, persons and entities are recognized both in
environment, which means that for the Petition of the human law and the Rules of Court as having standing to sue and, therefore,
petitioners to prosper, they must show that (a) the Resident Marine may be properly represented as real parties in interest. The same
Mammals are real parties in interest; and (b) that the human cannot be said about animals.
petitioners are authorized by law or the Rules to act in a
representative capacity. What may be argued as being parallel to this concept of
guardianship is the principle of human stewardship over the
The Resident Marine Mammals are comprised of "toothed environment in a citizen suit under the Rules of Procedure for
whales, dolphins, porpoises, and other cetacean species inhabiting Environmental Cases. A citizen suit allows any Filipino to act as a
Tañon Strait." While relatively new in Philippine jurisdiction, the issue representative of a party who has enforceable rights under
of whether animals have legal standing before courts has been the environmental laws before Philippine courts, and is defined in Section
subject of academic discourse in light of the emergence of animal and 5:
environmental rights.
SEC. 5. Citizen suit. - Any Filipino citizen in
In the United States, animal rights advocates have managed representation of others, including minors or
to establish a system which Hogan explains as the "guardianship generations yet unborn, may file an action to enforce
model for nonhuman animals": rights or obligations under environmental laws. Upon
the filing of a citizen suit, the court shall issue an order
“Despite Animal Lovers, there exists a well- which shall contain a brief description of the cause of
established system by which nonhuman animals may action and the reliefs prayed for, requiring all
obtain judicial review to enforce their statutory rights interested parties to manifest their interest to
and protections: guardianships. With court approval, intervene in the case within fifteen (15) days from
animal advocacy organizations may bring suit on notice thereof. The plaintiff may publish the order
behalf of nonhuman animals in the same way court- once in a newspaper of a general circulation in the
appointed guardians bring suit on behalf of mentally- Philippines or furnish all affected barangays copies of
said order.

43 | P a g e
CONSOLIDATED CASE DIGESTS in Political Law Review (1st Batch)

mere concern rather than an actual enforcement of a right. It is


There is no valid reason in law or the practical impossible for animals to tell humans what their concerns are. At
requirements of this case to implead and feign representation on best, humans can only surmise the extent of injury inflicted, if
behalf of animals. To have done so betrays a very there be any. Petitions invoking a right and seeking legal redress
anthropocentric view of environmental advocacy. There is no before this court cannot be a product of guesswork, and
way that we, humans, can claim to speak for animals let alone representatives have the responsibility to ensure that they bring
present that they would wish to use our court system, which is "reasonably cogent, rational, scientific, well-founded
designed to ensure that humans seriously carry their arguments"on behalf of those they represent.
responsibility including ensuring a viable ecology for
themselves, which of course includes compassion for all living Creative approaches to fundamental problems should be welcome.
things. However, they should be considered carefully so that no unintended
or unwarranted consequences should follow. Resident Marine
Our rules on standing are sufficient and need not be further Mammals and the human petitioners have no legal standing to file any
relaxed. kind of petition.
To revisit the ruling in Oposa v. Factoran:
26.) PORMENTO vs. ESTRADA (Judicial Review)
The use of the Oposa doctrine in environmental cases should FACTS: Private respondent was elected President of the Republic of
be allowed only when a) there is a clear legal basis for the the Philippines in the general elections held on May 11, 1998. He
representative suit; b) there are actual concerns based squarely upon sought the presidency again in the general elections held on May 10,
an existing legal right; c) there is no possibility of any countervailing 2010. Petitioner Atty. Evillo C. Pormento opposed private respondents
interests existing within the population represented or those that are candidacy and filed a petition for disqualification.
yet to be born; and d) there is an absolute necessity for such standing
because there is a threat of catastrophe so imminent that an However, his petition was denied by the Second Division of public
immediate protective measure is necessary. respondent Commission on Elections (COMELEC). His motion for
reconsideration was subsequently denied by the COMELEC en banc.
The Rules of Procedure for Environmental Cases allows filing
Petitioner filed the instant petition for certiorari on May 7, 2010.
of a citizen's suit. A citizen's suit under this rule allows any Filipino
However, under the Rules of Court, the filing of such petition would
citizen to file an action for the enforcement of environmental law on
not stay the execution of the judgment, final order or resolution of the
behalf of minors or generations yet unborn. It is essentially a
COMELEC that is sought to be reviewed. Besides, petitioner did not
representative suit that allows persons who are not real parties in
even pray for the issuance of a temporary restraining order or writ of
interest to institute actions on behalf of the real party in interest. It is
preliminary injunction. Hence, private respondent was able to
clear that jurisprudence only applies consistently to human petitioners
participate as a candidate for the position of President in the May 10,
including the unborn as represented by their human guardians.
2010 elections where he garnered the second highest number of
votes.
Extending the application of "real party in interest" to the
Resident Marine Mammals, or animals in general, through a judicial Private respondent was not elected President the second time he ran.
pronouncement will potentially result in allowing petitions based on

44 | P a g e
CONSOLIDATED CASE DIGESTS in Political Law Review (1st Batch)

ISSUE: WON Judicial review is applicable. Violence Against Women and Their Children, Providing for Protective
Measures for Victims, Prescribing Penalties Therefor, and for Other
HELD: NO! (THERE IS NO ACTUAL CONTROVERSY IN THE CASE Purposes.” She claimed to be a victim of physical, emotional,
AT BAR WHICH IS AN ESSENTIAL ELEMENT FOR JUDICIAL psychological and economic violence, being threatened of deprivation
REVIEW) of custody of her children and of financial support and also a victim of
There is no case or controversy to be resolved in this case. No marital infidelity on the part of petitioner.
live conflict of legal rights exists. There is in this case no definite, The TPO was granted but the petitioner failed to faithfully comply with
concrete, real or substantial controversy that touches on the the conditions set forth by the said TPO, private-respondent filed
legal relations of parties having adverse legal interests. No another application for the issuance of a TPO ex parte. The trial court
specific relief may conclusively be decreed upon by this Court in this issued a modified TPO and extended the same when petitioner failed
case that will benefit any of the parties herein. As such, one of the to comment on why the TPO should not be modified. After the given
essential requisites for the exercise of the power of judicial time allowance to answer, the petitioner no longer submitted the
review, the existence of an actual case or controversy, is sorely required comment as it would be an “exercise in futility.”
lacking in this case.
Petitioner filed before the CA a petition for prohibition with prayer for
As a rule, this Court may only adjudicate actual, ongoing injunction and TRO on, questioning the constitutionality of the RA
controversies.[9] The Court is not empowered to decide moot 9262 for violating the due process and equal protection clauses, and
questions or abstract propositions, or to declare principles or the validity of the modified TPO for being “an unwanted product of an
rules of law which cannot affect the result as to the thing in issue invalid law.”
in the case before it.
The CA issued a TRO on the enforcement of the TPO but however,
An action is considered moot when it no longer presents a denied the petition for failure to raise the issue of constitutionality in
justiciable controversy because the issues involved have his pleadings before the trial court and the petition for prohibition to
become academic or dead or when the matter in dispute has annul protection orders issued by the trial court constituted collateral
already been resolved and hence, one is not entitled to judicial attack on said law.
intervention unless the issue is likely to be raised again between
the parties. There is nothing for the court to resolve as the Petitioner filed a motion for reconsideration but was denied. Thus, this
determination thereof has been overtaken by subsequent events. petition is filed.

Following the results of that elections, private respondent was not Issues: WON the CA erred in dismissing the petition on the theory that
elected President for the second time. Thus, any discussion of his the issue of constitutionality was not raised at the earliest opportunity
reelection will simply be hypothetical and speculative. It will serve no and that the petition constitutes a collateral attack on the validity of the
useful or practical purpose. law.

27.) GARCIA vs. JUDGE DRILON WON the CA committed serious error in failing to conclude that RA
9262 is discriminatory, unjust and violative of the equal protection
FACTS: Private respondent Rosalie filed a petition before the clause.
RTC of Bacolod City a Temporary Protection Order against her
husband, Jesus, pursuant to R.A. 9262, entitled “An Act Defining

45 | P a g e
CONSOLIDATED CASE DIGESTS in Political Law Review (1st Batch)

WON the CA committed grave mistake in not finding that RA 9262 4. The non-referral of a VAWC case to a mediator is justified.
runs counter to the due process clause of the Constitution Petitioner’s contention that by not allowing mediation, the law violated
the policy of the State to protect and strengthen the family as a basic
WON the CA erred in not finding that the law does violence to the autonomous social institution cannot be sustained. In a memorandum
policy of the state to protect the family as a basic social institution of the Court, it ruled that the court shall not refer the case or any issue
WON the CA seriously erredin declaring RA 9262 as invalid and therof to a mediator. This is so because violence is not a subject for
unconstitutional because it allows an undue delegation of judicial compromise.
power to Brgy. Officials. 5. There is no undue delegation of judicial power to Barangay
HELD: 1. Petitioner contends that the RTC has limited authority officials. Judicial power includes the duty of the courts of justice to
and jurisdiction, inadequate to tackle the complex issue of settle actual controversies involving rights which are legally
constitutionality. Family Courts have authority and jurisdiction to demandable and enforceable and to determine whether or not there
consider the constitutionality of a statute. The question of has been a grave abuse of discretion amounting to lack or excess of
constitutionality must be raised at the earliest possible time so that if jurisdiction on any part of any branch of the Government while
not raised in the pleadings, it may not be raised in the trial and if not executive power is the power to enforce and administer the laws. The
raised in the trial court, it may not be considered in appeal. preliminary investigation conducted by the prosecutor is an executive,
not a judicial, function. The same holds true with the issuance of
2. RA 9262 does not violate the guaranty of equal protection of the BPO. Assistance by Brgy. Officials and other law enforcement
laws. Equal protection simply requires that all persons or things agencies is consistent with their duty executive function.
similarly situated should be treated alike, both as to rights conferred
and responsibilities imposed. In Victoriano v. Elizalde Rope The petition for review on certiorari is denied for lack of merit
Workerkers’ Union, the Court ruled that all that is required of a valid 28.) SOCIAL JUSTIC SOCIETY (SJS) vs. LIM
classification is that it be reasonable, which means that the
classification should be based on substantial distinctions which make (G.R. Nos. 187836 and 187916)
for real differences; that it must be germane to the purpose of the law;
not limited to existing conditions only; and apply equally to each 25 November 2014
member of the class. Therefore, RA9262 is based on a valid FACTS: On 12 October 2001, a Memorandum of Agreement was
classification and did not violate the equal protection clause by entered into by oil companies (Chevron, Petron and Shell) and
favouring women over men as victims of violence and abuse to whom Department of Energy for the creation of a Master Plan to address and
the Senate extends its protection. minimize the potential risks and hazards posed by the proximity of
3. RA 9262 is not violative of the due process clause of the communities, business and offices to Pandacan oil terminals without
Constitution. The essence of due process is in the reasonable affecting security and reliability of supply and distribution of petroleum
opportunity to be heard and submit any evidence one may have in products.
support of one’s defense. The grant of the TPO exparte cannot be On 20 November 2001, the Sangguniang Panlungsod (SP) enacted
impugned as violative of the right to due process. Ordinance No. 8027 which reclassifies the land use of Pandacan, Sta.
Ana, and its adjoining areas from Industrial II to Commercial I. Owners
and operators of the businesses affected by the reclassification were

46 | P a g e
CONSOLIDATED CASE DIGESTS in Political Law Review (1st Batch)

given six (6) months from the date of effectivity to stop the operation Their contentions are as follows: It is an invalid exercise of police
of their businesses. It was later extended until 30 April 2003. · On 4 power because it does not promote the general welfare of the people
December 2002, a petition for mandamus was filed before the It is violative of Section 15 and 16, Article II of the 1987 Constitution
Supreme Court (SC) to enforce Ordinance No. 8027. Unknown to the as well as health and environment related municipal laws and
SC, the oil companies filed before the Regional Trial Court of Manila international conventions and treaties, such as: Clean Air Act;
an action to annul Ordinance No. 8027 with application for writs of Environment Code; Toxic and Hazardous Wastes Law; Civil Code
preliminary prohibitory injunction and preliminary mandatory provisions on nuisance and human relations; Universal Declaration of
injunction. The same was issued in favor of Chevron and Shell. Human Rights; and Convention on the Rights of the Child · The title
Petron, on the other hand, obtained a status quo on 4 August 2004. of Ordinance No. 8187 purports to amend or repeal Ordinance No.
8119 when it actually intends to repeal Ordinance No. 8027. On the
On 16 June 2006, Mayor Jose Atienza, Jr. approved Ordinance No. other hand, the respondents Mayor Lim, et.al. and the intervenors oil
8119 entitled †œAn Ordinance Adopting the Manila Comprehensive companies contend that: The petitioners have no legal standing to sue
Land Use Plan and Zoning Regulations of 2006 and Providing for the whether as citizens, taxpayers or legislators. They further failed to
Administration, Enforcement and Amendment thereto. This show that they have suffered any injury or threat of injury as a result
designates Pandacan oil depot area as a Planned Unit of the act complained of· The petition should be dismissed outright for
Development/Overlay Zone. failure to properly apply the related provisions of the Constitution, the
On 7 March 2007, the SC granted the petition for mandamus and Rules of Court, and/or the Rules of Procedure for Environmental
directed Mayor Atienza to immediately enforce Ordinance No. 8027. Cases relative to the appropriate remedy available.
It declared that the objective of the ordinance is to protect the residents The principle of the hierarchy of courts is violated because the SC only
of manila from the catastrophic devastation that will surely occur in exercises appellate jurisdiction over cases involving the
case of a terrorist attack on the Pandacan Terminals. constitutionality or validity of an ordinance under Section 5, Article VIII
The oil companies filed a Motion for Reconsideration (MR) on the 7 of the 1987 Constitution. It is the function of the SP to enact zoning
March 2007 Decision. The SC later resolved that Ordinance No. 8027 ordinance without prior referral to the Manila Zoning Board of
is constitutional and that it was not impliedly repealed by Ordinance Adjustment and Appeals; thus, it may repeal all or part of zoning
No. 8119 as there is no irreconcilable conflict between them. SC later ordinance sought to be modified. There is a valid exercise of police
on denied with finality the second MR of the oil companies. power.

On 14 May 2009, during the incumbency of Mayor Alfredo Lim (Mayor On 28 August 2012, the SP enacted Ordinance No. 8283 which
Lim), the SP enacted Ordinance No. 8187. The Industrial Zone under essentially amended the assailed Ordinance to exclude the area
Ordinance No. 8119 was limited to Light Industrial Zone, Ordinance where petroleum refineries and oil depots are located from the
No. 8187 appended to the list a Medium and a Heavy Industrial Zone Industrial Zone. The same was vetoed by Mayor Lim.
where petroleum refineries and oil depots are expressly allowed. ISSUES:
Petitioners Social Justice Society Officers, Mayor Atienza, et.al. filed 1. WON there are violations of environmental laws
a petition for certiorari under Rule 65 assailing the validity of
Ordinance No. 8187. 2. WON the principle of hierarchy of courts is violated

47 | P a g e
CONSOLIDATED CASE DIGESTS in Political Law Review (1st Batch)

3. WON the petitioners have legal standing to sue a direct interest in the implementation of the ordinances of the city. No
different are herein petitioners who seek to prohibit the enforcement
4. WON Ordinance No. 8187 is unconstitutional in relation to the of the assailed ordinance, and who deal with the same subject matter
Pandacan Terminals that concerns a public right. In like manner, the preservation of the life,
RULING: security and safety of the people is indisputably a right of utmost
importance to the public. Certainly, the petitioners, as residents of
1. None. The scope of the Rules of Procedure for Environmental Manila, have the required personal interest to seek relief to protect
Cases is embodied in Section 2, Part I, Rule I thereof. It states that the such right.
Rules shall govern the procedure in civil, criminal and special civil
actions before the MeTCs, MTCCs, MTCs and MCTCs, and the RTCs 4. Yes. In striking down the contrary provisions of the assailed
involving the enforcement or violations of environmental and other Ordinance relative to the continued stay of the oil depots, the SC
related laws, rules and regulations such as but not limited to: R.A. No. followed the same line of reasoning used in its 7 March 2007 decision,
6969, Toxic Substances and Hazardous Waste Act; R.A. No. 8749, to wit: Ordinance No. 8027 was enacted for the purpose of promoting
Clean Air Act; Provisions in C.A. No. 141; and other existing laws that a sound urban planning, ensuring health, public safety and general
relate to the conservation, development, preservation, protection and welfare of the residents of Manila. The Sanggunian was impelled to
utilization of the environment and natural resources. Notably, the take measures to protect the residents of Manila from catastrophic
aforesaid Rules are limited in scope. While, indeed, there are devastation in case of a terrorist attack on the Pandacan Terminals.
allegations of violations of environmental laws in the petitions, these Towards this objective, the Sanggunian reclassified the area defined
only serve as collateral attacks that would support the other position in the ordinance from industrial to commercial. The following facts
of the petitioners, the protection of the right to life, security and safety. were found by the Committee on Housing, Resettlement and Urban
2. No. The SC held that it is true that the petitions should have been Development of the City of Manila which recommended the approval
filed with the RTC, it having concurrent jurisdiction with the SC over a of the ordinance:
special civil action for prohibition, and original jurisdiction over (1) The depot facilities contained 313.5 million liters of highly
petitions for declaratory relief. However, the petitions at bar are of flammable and highly volatile products which include petroleum gas,
transcendental importance warranting a relaxation of the doctrine of liquefied petroleum gas, aviation fuel, diesel, gasoline, kerosene and
hierarchy of courts. This is in accordance with the well-entrenched fuel oil among others;
principle that rules of procedure are not inflexible tools designed to
hinder or delay, but to facilitate and promote the administration of (2) The depot is open to attack through land, water and air;
justice. Their strict and rigid application, which would result in
technicalities that tend to frustrate, rather than promote substantial (3) It is situated in a densely populated place and near Malacanang
justice, must always be eschewed. (Jaworski v. PAGCOR, 464 Phil. Palace; and
375) (4) In case of an explosion or conflagration in the depot, the fire could
3. Yes. The SC referred to their Decision dated 7 March 2007 which spread to the neighboring communities. The Ordinance was intended
ruled that the petitioners in that case have a legal right to seek the to safeguard the rights to life, security and safety of all the inhabitants
enforcement of Ordinance No. 8027 because the subject of the of Manila and not just of a particular class. The depot is perceived,
petition concerns a public right, and they, as residents of Manila, have rightly or wrongly, as a representation of western interests which

48 | P a g e
CONSOLIDATED CASE DIGESTS in Political Law Review (1st Batch)

means that it is a terrorist target. As long as there is such a target in on fair market value and not that set by Presidential Decree No. 76,
their midst, the residents of Manila are not safe. It therefore becomes as amended, which fixes payment on the basis of the assessment by
necessary to remove these terminals to dissipate the threat. the assessor or the declared valuation by the owner, whichever is
lower. The RTC ruled for the private respondents. The CA affirmed
The same best interest of the public guides the present decision. The said decision. Hence, the instant petition by the Republic.
Pandacan oil depot remains a terrorist target even if the contents have
been lessened. In the absence of any convincing reason to persuade In Export Processing Zone Authority (“EPZA”) vs. Dulay, etc. et al., the
the Court that the life, security and safety of the inhabitants of Manila Court held the determination of just compensation in eminent domain
are no longer put at risk by the presence of the oil depots, the SC holds to be a judicial function and it thereby declared Presidential Decree
that the Ordinance No. 8187 in relation to the Pandacan Terminals is No. 76, as well as related decrees, including Presidential Decree No.
invalid and unconstitutional. For, given that the threat sought to be 1533, to the contrary extent, as unconstitutional and as an
impermissible encroachment of judicial prerogatives. The ruling, now
prevented may strike at one point or another, no matter how remote it
conceded by the Republic was reiterated in subsequent cases.
is as perceived by one or some, the SC cannot allow the right to life
be dependent on the unlikelihood of an event. Statistics and theories ISSUE: Whether the declaration of nullity of the law in question should
of probability have no place in situations where the very life of not just have prospective, not retroactive, application.
an individual but of residents of big neighbourhoods is at stake.
HELD: There are two views on the effects of a declaration of the
DISPOSITIVE PORTION unconstitutionality of a statute:
1. Ordinance No. 8187 is declared unconstitutional and invalid with
The first is the orthodox view. Under this rule, as announced in Norton
respect to the continued stay of the Pandacan Oil Terminals.
v. Shelby, an unconstitutional act is not a law; it confers no right; it
2. The incumbent mayor of the City of Manila is ordered to cease and imposes no duties; it affords no protection; it creates no office; it is, in
desist from enforcing Ordinance No. 8187 and to oversee the legal contemplation, inoperative, as if it had not been passed. It is
relocation and transfer of the oil terminals out of the Pandacan area therefore stricken from the statute books and considered never to
have existed at all. Not only the parties but all persons are bound by
3. The oil companies shall, within a non-extendible period of forty-five the declaration of unconstitutionality, which means that no one may
(45) days, submit to the RTC Manila, Branch 39 an updated thereafter invoke it nor may the courts be permitted to apply it in
comprehensive plan and relocation schedule, which relocation shall subsequent cases. It is, in other words, a total nullity.
be completed not later than six (6) months from the date the required
The second or modern view is less stringent. Under this view, the
document is submitted.
court in passing upon the question of constitutionality does not annul
29.) REPUBLIC OF THE PHILIPPINES vs. CA, HENRICO or repeal the statute if it finds it in conflict with the Constitution. It simply
UVERO, ET AL. refuses to recognize it and determines the rights of the parties just as
if such statute had no existence. The court may give its reasons for
G.R. No. 79732 November 8, 1993 ignoring or disregarding the law, but the decision affects the parties
only and there is no judgment against the statute. The opinion or
FACTS: The Republic of the Philippines has sought the expropriation reasons of the court may operate as a precedent for the determination
of certain portions of land owned by the private respondents. The latter of other similar cases, but it does not strike the statute from the statute
demand that the just compensation for the property should be based books; it does not repeal, supersede, revoke, or annul the statute. The

49 | P a g e
CONSOLIDATED CASE DIGESTS in Political Law Review (1st Batch)

parties to the suit are concluded by the judgment, but no one else is
bound.

The orthodox view is expressed in Article 7 of the Civil Code, providing


that “when the courts declare a law to be inconsistent with the
Constitution, the former shall be void and the latter shall govern. . . .

An otherwise valid law may be held unconstitutional only insofar as it


is allowed to operate retrospectively such as, in pertinent cases, when
it vitiates contractually vested rights. To that extent, its retroactive
application may be so declared invalid as impairing the obligations of
contracts. A judicial declaration of invalidity, it is also true, may not
necessarily obliterate all the effects and consequences of a void act
occurring prior to such a declaration.

The fact of the matter is that the expropriation cases, involved in


this instance, were still pending appeal when the EPZA ruling
was rendered and forthwith invoked by said parties. The appellate
court in this particular case committed no error in its appealed
decision. The instant petition is dismissed.

50 | P a g e

S-ar putea să vă placă și